MBE Questions Flashcards by Abby Schwarz (2024)

1

Q

The defendant is being tried for murder in the bludgeoning death of his brother. The defendant denies any involvement in the crime. He callsa witness to the stand, who testifies that, in his opinion, the defendant is a nonviolent, peaceable man.

Which of the following, if offered by the prosecution, would most likely be admissible?

(A) A neighbor’s testimony that the witness has beaten his wife on several occasions.
(B) A police officer’s testimony that the defendant has a general reputation in the community as a violent person.
(C) A neighbor’s testimony that the defendant has a reputation for being untruthful.
(D) Evidence that the defendant has a conviction for aggravated battery.

A

(B) A police officer’s testimony that the defendant has a general reputation in the community as a violent person.

Criminal = Reputation, Opinion ONLY
1) Did D open the door? YES
2) Is the character trait relevant? YES (violence)

The defendant put his character of nonviolence at issue by having his witness testify to the defendant’s nonviolent nature, which is relevant to whether he committed the crime charged.

Note: The D’s reputation for truthfulness is not relevant to whether he has committed the crime for which
he is charged. (And because the defendant has not placed his credibility in issue by taking the stand as a witness, his reputation for truthfulness cannot be offered for impeachment purposes.)

How well did you know this?

1

Not at all

2

3

4

5

Perfectly

2

Q

A lawyer represented a sole proprietor in a federal income tax refund case in U.S. district court in which the sole proprietor was claiming certain additional business expense deductions based on sales expenses that had inadvertently been left off of his tax return for the year.

What is most likely protected from admission into evidence under the attorney-client

A) An employment contract between the sole proprietor and a salesperson drafted by the lawyer.
B) Sales records created by the sole proprietor, and provided to the lawyer to enable the lawyer to prepare for the litigation.
C) A letter sent to the lawyer by the sole proprietor detailing business expenses.
D) The sole proprietor’s tax return for the tax year in question, which was prepared by the lawyer.

A

A letter sent to the lawyer by the sole proprietor detailing business expenses.

The letter is a communication by a client to his attorney about a matter under litigation.

Note: The letter protected by the attorney-client privilege, the information contained in the letter, i.e., the sole proprietor’s business expenses, is not protected by this privilege

Note: B is incorrect because, even though the records were provided by the client to the lawyer and pertained to the legal matter at hand, the records were not prepared in order to obtain legal advice.

How well did you know this?

1

Not at all

2

3

4

5

Perfectly

3

Q

A telecommunications company retained a lawyer to represent it in a commercial dispute in which it was the plaintiff. The fee agreement provided that the lawyer would bill the company on an hourly basis but would not collect anything except costs “unless and until the telecommunications company received a recovery or settlement in the action.”

Due to disagreements with the lawyer’s strategy for the trial, the telecommunications company discharged the lawyer and hired a new law firm on a contingency basis to represent it in the case. The company was ultimately awarded a large judgment.

The original lawyer sued the company for her uncollected fees. The company denied owing the lawyer anything, asserting that no money was due yet because the judgment had not actually been collected. The lawyer wants to introduce evidence that the judgment has been collected by showing disbursem*nts from the new law firm’s trust account to the telecommunications company.

Is this evidence admissible?

A) No, because the evidence is protected by the attorney-client privilege.
B) No, because the evidence is irrelevant.
C) Yes, because the evidence does not constitute a confidential communication covered by the attorney-client privilege.
D) Yes, because the attorney-client privilege does not apply to disputes between the client and the attorney.

A

C) Yes, because the evidence does not constitute a confidential communication covered by the attorney-client privilege.

The attorney-client privilege prevents anyone from testifying about confidential communications made to an attorney for the purpose of obtaining legal services.

BUT the payments made out of the trust account were not communications made for the purpose of obtaining representation, and are therefore not covered by this privilege.

Note: Although there is an exception to attorney-client privilege when there is a dispute between the attorney and the client, the dispute at issue is between the company and the original lawyer, not the company and the law firm that made this disbursem*nt. Moreover, the payments made out of the trust account were not communications made for the purpose of obtaining representation, and are therefore not covered by this privilege.

How well did you know this?

1

Not at all

2

3

4

5

Perfectly

4

Q

A governmental inspector of a construction project was struck and injured by a truck. The inspector brought a negligence action against the construction firm, alleging that the driver of the truck was an employee of the firm. At trial, the head of the construction firm testified that the truck driver was an independent contractor. The inspector’s lawyer, in cross-examining the head of the construction firm, sought to ask whether the construction firm had purchased liability insurance for the truck.

Can the judge permit this question over the objection of the lawyer for the construction firm?

A) No, because evidence of liability insurance is inadmissible in a negligence action.
B) No, because whether the construction firm had liability insurance can only be proved by extrinsic evidence.
C) Yes, because evidence of liability insurance is subject to disclosure during discovery under the Federal Rules of Civil Procedure.
D) Yes, because the evidence tends to show that the truck driver was an employee of the construction firm.

A

D) Yes, because the evidence tends to show that the truck driver was an employee of the construction firm.

Other purpose: proving control

How well did you know this?

1

Not at all

2

3

4

5

Perfectly

5

Q

A plaintiff sued a defendant for injuries he suffered while shaving with a razor manufactured by the defendant. The plaintiff alleged that the razor was defectively designed. The plaintiff’s expert witness testified that the manufacturer should have used certain safeguards in the razor’s design that would have made the razor safer. In his testimony, the defendant did not deny that the safeguards urged by the plaintiff’s expert were feasible, but he argued that they were unnecessary and that the razor was not defectively designed. The plaintiff seeks to cross-examine the defendant about a safety modification the defendant has made to the razor since the plaintiff’s injury.

For which of the following substantive purposes may evidence of this modification be used?

A) Both to prove that the razor was defectively designed and to prove that the safeguards described by the expert were feasible.
B) Only to prove that the razor was defectively designed.
C) Only to prove that the safeguards described by the expert were feasible.
D) Neither to prove that the razor was defectively designed nor to prove that the safeguards described by the expert were feasible.

A

D) Neither to prove that the razor was defectively designed nor to prove that the safeguards described by the expert were feasible.

Plaintiff may not introduce evidence of remedial measure – feasibility unless the feasibility of such measures is disputed

How well did you know this?

1

Not at all

2

3

4

5

Perfectly

6

Q

A woman took out a loan from a large bank in order to start a new business. After she missed several payments, the bank sued the woman to collect the outstanding balance. In pre-trial settlement negotiations, the woman explained that she was unable to make the payments because her business was struggling. She noted that she should have listened to her boyfriend, who believed that debt would be the downfall of the country and that people should do whatever necessary to destroy big banks.

The parties eventually reached a compromise whereby the bank would extend the time to repay the loan, but would retain the right to sue under the original terms if the woman missed any payments. When the woman failed to make the payments, the bank properly reinstituted its suit against the woman.

At trial, the woman claimed that the loan contract was invalid because she was heavily medicated at the time she made the contract, and was thus incompetent. The woman has called her boyfriend to the stand to testify to these facts. The bank seeks to introduce the woman’s statements about her boyfriend made during the earlier settlement negotiations.

Are the woman’s statements about her boyfriend likely to be admitted?

A) No, because the statements were made during settlement negotiations.
B) No, because the statements are inadmissible hearsay.
C) Yes, because the statements prove bias or prejudice of a witness.
D) because the negotiations at issue resulted in a settlement agreement.

A

C) Yes, because the statements prove bias or prejudice of a witness.

Statements made during settlement negotiations are inadmissible to prove or disprove the validity or amount of a disputed claim. Such statements may be admitted for other purposes, however, such as to prove the bias or prejudice of a witness. In this case, the bank intends to introduce the woman’s statements about her boyfriend to show his bias against banks. Accordingly, they are admissible even though they were made during a settlement negotiation.

How well did you know this?

1

Not at all

2

3

4

5

Perfectly

7

Q

A customer sued a home improvement store for damages due to an alleged back injury that occurred when an employee driving a forklift backed into the unsuspecting customer who was facing the opposite direction. The store asserted that the forklift accident had not caused the customer’s injury. At trial, the customer seeks to introduce an affidavit of a physician, who has since died, that she examined the customer the day after the incident and concluded that the customer had suffered a back injury within the past 36 hours. Is this affidavit admissible?

A) No, because of the physician-patient privilege.
B) No, because it is inadmissible hearsay.
C) Yes, because it is recorded former testimony.
D) Yes, because it is a statement regarding a present physical condition.

A

B) No, because it is inadmissible hearsay.

Out of court statement? Yes
Used for the truth of the matter asserted? Yes

Is it hearsay (exemptions)? No
Exceptions? No

Note: * statement was not made by the customer about his own physical condition but by the physician – medical treatment exception does not apply

How well did you know this?

1

Not at all

2

3

4

5

Perfectly

8

Q

In a murder trial, the prosecutor planned to call an eyewitness to the stand to testify that he saw the defendant kill the victim. However, the witness recently suffered a severe head injury that seriously affected his memory. The witness can no longer remember witnessing the murder. Prior to the witness’s injury, he testified to what he saw before the grand jury.

The prosecutor would like to introduce the witness’s grand jury testimony as substantive evidence that the defendant committed the murder. The defendant objects to the introduction of the evidence. Should the court admit the witness’s grand jury testimony into evidence?

A) Yes, because the witness is unavailable to testify.
B) if used to refresh the witness’s recollection.
C) No, because the witness does not meet the “unavailability” standard.
D) No, because the former testimony exception does not apply to these facts.

A

D) No, because the former testimony exception does not apply to these facts.

Although the witness is “unavailable” for the purposes of the hearsay rules (as will be discussed below), and there is a “former testimony” exception to the hearsay rule, the former testimony exception does not apply to grand jury testimony.

Note that this WOULD be OK for prior inconsistent statement, but that’s not happening here.

How well did you know this?

1

Not at all

2

3

4

5

Perfectly

9

Q

A defendant was sued in civil court for assault. The defendant, as his first witness in his case in chief, called a friend to testify that, on the day before the day in question, the defendant had told her that he was leaving town that afternoon to drive across the country.

Is this testimony admissible to show that the defendant was not in town when the assault allegedly occurred?

A) Yes, because the defendant is available to testify.
B) Yes, because it is a declaration of the defendant’s present mental state.
C) No, because it is hearsay.
D) No, because the witness is biased.

A

B) Yes, because it is a declaration of the defendant’s present mental state.

A statement of present intent, motive, or plan can be used to prove conduct in conformity with that state of mind. Consequently, the defendant’s statement to his friend may be introduced into evidence to show that the defendant was not in town when the assault allegedly occurred.

How well did you know this?

1

Not at all

2

3

4

5

Perfectly

10

Q

A woman and her sister walked into the woman’s house. The woman went into her bedroom to say hello to her husband, while the sister waited in the other room. As soon as the woman walked into the bedroom, her sister heard her exclaim, “Where did you get all that money? There must be several thousand dollars on this dresser!” The woman came out of her bedroom shortly thereafter, and told her sister that the sister should leave immediately. That night, the woman called her sister and told her that her husband had robbed a bank. The husband was later arrested for bank robbery. At trial, the woman refused to testify against her husband, and the prosecution called the sister as a witness. The sister testified that the woman had said there was several thousand dollars on the dresser, and that the woman called her to tell her that the husband had robbed a bank. The husband objected to both pieces of testimony.

How should the judge rule regarding the sister’s testimony?

A) The judge should admit both pieces of testimony.
B) The judge should admit the testimony regarding the woman’s statement about the money on the dresser only.
C) The judge should admit the testimony regarding the woman’s statement about her husband robbing a bank only.
D) The judge should exclude both pieces of testimony.

A

B) The judge should admit the testimony regarding the woman’s statement about the money on the dresser only.

Excited Utterance/Present Sense Impression: woman made the statement immediately upon noticing the money on the dresser.

No exception for the later phone call.

How well did you know this?

1

Not at all

2

3

4

5

Perfectly

11

Q

A defendant was on trial for having committed a murder in 1995. Taking the stand, the defendant denied being present in the city where the murder occurred at the time of the killing. The prosecution sought to admit into evidence a copy of the local newspaper published the day after the killing. The newspaper contained an article in which the defendant was quoted as stating that he had heard shots on the day of the murder from inside his apartment. The defendant objected to the introduction of this evidence. Should the judge admit the newspaper article into evidence?

A) Yes, both to impeach the defendant’s credibility and to prove that he was in the city on the day of the murder.
B) Yes, but only for the purposes of impeaching the defendant’s credibility.
C) No, because the article is hearsay not within any exception.
D) No, because the article was not properly authenticated.

A

A) Yes, both to impeach the defendant’s credibility and to prove that he was in the city on the day of the murder.

Is it hearsay? No, inconsistent statement and opposing party statement

Newspaper exception: Ancient document
quote exception

How well did you know this?

1

Not at all

2

3

4

5

Perfectly

12

Q

In the prosecution of a defendant for murder, the state seeks to qualify a forensic analyst as an expert in order to have her testify as to her professional opinion of the crime scene. The defense has objected on the ground of inadequate qualifications. The prosecution now seeks to introduce a letter written by the editor-in-chief of a well-respected academic journal of forensic science, stating that the forensic analyst has published a number of well-reviewed papers on the subject of crime-scene analysis and is generally acknowledged in her field as very qualified.

On the issue of the forensic analyst’s qualifications, may the judge consider the editor’s letter?

A) Yes, because the letter is not hearsay.
B) Yes, because the judge may consider the letter without regard for the hearsay rule.
C) No, because the letter is hearsay not within any exception.
D) No, because it is the role of the jury to determine the credibility of the evidence of the forensic analyst’s qualifications.

A

B) Yes, because the judge may consider the letter without regard for the hearsay rule.

Whether a witness is qualified to offer expert opinion testimony is a preliminary question for the court—not the jury. The court is not bound by the rules of evidence when determining such questions.

How well did you know this?

1

Not at all

2

3

4

5

Perfectly

13

Q

A jurisdiction defines receiving stolen property as (i) receiving control of stolen property, (ii) with the knowledge that the property is stolen, and (iii) with the intent to permanently deprive the owner of the property. A defendant, charged with receiving stolen property after the police found a stolen television in his home, denied that he knew it was stolen. On cross-examination, the prosecutor asked the defendant, “Didn’t you also previously buy a stolen stereo from the same man who sold you this television?” The defendant’s attorney immediately objected.

What is the strongest basis for the defense attorney’s objection?

A) The probative value of the prosecutor’s question is substantially outweighed by the danger of unfair prejudice.
B) The prosecutor’s question was irrelevant because it does not establish an element that the prosecutor must prove.
C) The relevance of the prosecutor’s question depends upon whether the defendant knew the stereo was stolen, and the prosecutor has not offered sufficient proof to support that finding.
D) The risk of unfair prejudice is not substantially outweighed by the probative value of the prosecutor’s question.

A

A) The probative value of the prosecutor’s question is substantially outweighed by the danger of unfair prejudice.

A court may exclude relevant evidence when its probative value is substantially outweighed by a danger of unfair prejudice, confusing the issues, misleading the jury, undue delay, wasting time, or needless cumulation of evidence.

Note: An FRE 403 objection requires that the court decide if the proffered evidence’s probative value is substantially outweighed by one of the above-listed dangers—not vice versa.

How well did you know this?

1

Not at all

2

3

4

5

Perfectly

14

Q

In his criminal trial for battery, a defendant sought to introduce evidence of his peaceful character. He had met the leader of a local animal rights group once during a recent animal rights demonstration in front of the mayor’s office, during which the defendant succeeded in calming down an angry group of protestors. The defendant planned to ask the group leader to testify about this incident to the jury. The prosecution objected to the introduction of this evidence.

How should the court rule on the objection?

A) Sustain the objection, because this testimony constitutes an inappropriate use of character evidence.
B) Sustain the objection, because the leader of the animal rights group only met the defendant once.
C) Overrule the objection, because the defendant may introduce evidence of his good character if relevant to the crime charged.
D) Overrule the objection, because specific acts are admissible in criminal cases if introduced by the defendant.

A

A) Sustain the objection, because this testimony constitutes an inappropriate use of character evidence.

A criminal defendant may introduce evidence that his/her character is inconsistent with the crime charged. But the defendant may only do so through reputation or opinion testimony—not specific acts of conduct.

How well did you know this?

1

Not at all

2

3

4

5

Perfectly

15

Q

A politician on trial for the misdemeanor assault and battery of a reporter asserts that the reporter started the altercation by shouting questions in his face and shoving him. At trial, the politician did not take the stand and did not introduce evidence of his own character, but he did call the reporter’s neighbor to testify that the reporter has a reputation among neighbors for violent outbursts. After the neighbor testified, the prosecution moved to introduce testimony by a community leader that the politician has a reputation for violence in the community as evidence that the politician started the altercation.

Is the prosecution’s evidence regarding the politician’s reputation admissible?

A) No, because the politician did not “open the door” to the introduction of evidence of his bad character by introducing evidence of his own good character.
B) No, because the politician’s character is not at issue, as he did not testify.
C) Yes, because the politician “opened the door” to the evidence of his bad character for violence by introducing evidence of the reporter’s character for violence.
D) Yes, because violence is an essential element of battery.

A

C) Yes, because the politician “opened the door” to the evidence of his bad character for violence by introducing evidence of the reporter’s character for violence.

A criminal defendant opens the door for the prosecution to introduce evidence of the defendant’s bad character by introducing:
(1) evidence of his/her own good character for a trait pertinent to the charged crime or
(2) evidence of the alleged victim’s bad character.

How well did you know this?

1

Not at all

2

3

4

5

Perfectly

16

Q

A defendant was charged with battery following a bar fight with his neighbor. At trial, the defendant asserted that he did not initiate the altercation, but instead acted in self-defense. In addition to testifying about the event in question, he sought to testify that the preceding night, he and a coworker had gone out for a drink at the same bar, and that the evening had passed peacefully. Prior to his own testimony, the defendant sought to introduce testimony of a lifelong acquaintance of the neighbor that, in the opinion of the acquaintance, the neighbor had a violent streak. After testifying, the defendant sought to introduce testimony of the pastor of the church that the defendant regularly attended that the defendant had a reputation among the members of the church as a nonviolent person. Following testimony introduced by the prosecution that impeached the defendant’s truthfulness, the defendant sought to introduce testimony of his employer that, in his opinion, the defendant was a truthful individual.

Which of the proffered testimony is most likely to be successfully challenged by the prosecution?

A) The testimony of the lifelong acquaintance of the neighbor regarding the neighbor’s violent streak.
B) The testimony of the defendant regarding his peaceful behavior on the night before the bar fight.
C) The testimony of the defendant’s pastor as to the defendant’s reputation as a nonviolent person.
D) The testimony of the defendant’s employer that the defendant was a truthful individual.

A

B) The testimony of the defendant regarding his peaceful behavior on the night before the bar fight.

A criminal defendant may introduce evidence that his character is inconsistent with the crime charged, but only through reputation or opinion testimony—not specific instances of conduct.

How well did you know this?

1

Not at all

2

3

4

5

Perfectly

17

Q

A defendant was charged with burglary. One of the key pieces of evidence in the case was a note left by the burglar that read, “It’s just 2 easy.” At trial, the defendant testified in his defense, asserting that he did not commit the crime. On cross-examination, the prosecutor, having a proper factual basis, asked the defendant if he had been convicted of felony burglary five years ago after having left a note at the crime scene that read, “It’s just 2 easy.” The defendant’s attorney, having received proper notice from the prosecutor regarding use of the prior conviction, objected to the prosecutor’s question as seeking to elicit improper criminal-propensity evidence. The court, after determining that the probative value of this evidence and its prejudicial effects were equal, overruled the objection and instructed the defendant to answer the question.

Has the court acted properly?

A) No, because evidence of the defendant’s prior conviction constitutes improper criminal-propensity evidence.
B) No, because the court did not find that the probative value of the conviction outweighed its prejudicial effects.
C) Yes, because the defendant may be impeached by a prior conviction of burglary within the last 10 years.
D) Yes, because the prior conviction helps establish the defendant as the perpetrator of the burglary for which he is on trial.

A

D) Yes, because the prior conviction helps establish the defendant as the perpetrator of the burglary for which he is on trial.

Evidence of a criminal defendant’s prior crimes or bad acts may be admissible for relevant, noncharacter purposes (i.e., MIMIC evidence). However, this and other relevant evidence may be excluded if its probative value is substantially outweighed by the danger of unfair prejudice.

How well did you know this?

1

Not at all

2

3

4

5

Perfectly

18

Q

A defendant was charged with aggravated assault arising from an altercation following a car accident. The defendant called a witness who testified that, in his opinion, the defendant was a nonviolent person. On cross-examination, the prosecutor asked the witness whether he was aware that the defendant had been involved in a bar fight during the past year. Although the defendant was actually involved in such a fight, the prosecutor herself was not aware of the incident, as the defendant had not been arrested or charged. However, the prosecutor had witnessed the defendant’s short temper during her interactions with the defendant and knew that he was a drinker. As it happened, the witness was aware of the defendant’s involvement in the fight.

Should the court require the witness to answer the prosecutor’s question?

A) Yes, because the witness’s knowledge of defendant’s past behavior goes to the witness’s credibility.
B) Yes, as a specific instance of the defendant’s conduct.
C) No, because the defendant was not arrested or charged in connection with the bar fight.
D) No, because the prosecution did not know the defendant had been involved in a bar fight.

A

D) No, because the prosecution did not know the defendant had been involved in a bar fight.

A criminal defendant may call a witness to testify that the defendant’s character is inconsistent with the charged crime.

The prosecution may then
(1) ask the witness about a specific act committed by the defendant or
(2) call another witness to provide reputation or opinion testimony on the defendant’s corresponding bad-character trait.

Here, the prosecutor asked the defendant’s character witness about a bar fight (specific act) the defendant had within the past year. This question would affect the witness’s credibility because it raises a doubt about the witness’s opinion that the defendant was a nonviolent person. But since the prosecution did not know the defendant had been involved in a bar fight, she asked the question on a hunch. Therefore, the question was not asked in good faith, and the court should not require the witness to answer it

How well did you know this?

1

Not at all

2

3

4

5

Perfectly

19

Q

A defendant was charged with assault. Upon learning that the defendant intended to testify in his own defense, the government gave the defense proper notice of its intent to introduce as impeachment evidence the defendant’s conviction for embezzlement nine years prior. The defense filed a motion to exclude all evidence of the defendant’s conviction, arguing that it would prejudice the defendant. In a pretrial hearing, the judge noted that the conviction would likely have little prejudicial effect.

Is the judge likely to grant the defendant’s motion?

A) No, because the conviction relates to a crime involving dishonesty and occurred within the last 10 years.
B) No, because the probative value of such a conviction outweighs its prejudicial effect.
C) Yes, because a prior conviction may not be used to impeach a defendant who testifies in his own defense.
D) Yes, because the conviction for embezzlement is not probative in determining whether the defendant committed an assault.

A

A) No, because the conviction relates to a crime involving dishonesty and occurred within the last 10 years.

Any witness can be impeached with evidence of a prior conviction for a crime involving dishonesty (e.g., embezzlement) if the conviction occurred within the previous 10 years.

Note: The embezzlement conviction is not probative in determining whether the defendant committed an assault. But the conviction may still be used to impeach the defendant’s character for truthfulness

How well did you know this?

1

Not at all

2

3

4

5

Perfectly

20

Q

A defendant is on trial for bank robbery. In seeking to prove that the defendant was the robber, the prosecution introduced a handwritten note given by the robber to the bank teller on her first day of work. The teller testified that the note presented to her on the witness stand was the note that she had received from the robber. The prosecution also seeks to have the teller testify as a lay witness that the handwriting on the note is that of the defendant, who was a bank customer, based on her comparison of the note with 10 customer signature cards, including the defendant’s, presented to her by the prosecutor after the robbery.

Is the teller’s testimony that the handwriting on the note matches that on the defendant’s customer signature card admissible?

A) No, because a lay witness may not testify as to whether a document is in a person’s handwriting.
B) No, because the teller’s familiarity with the defendant’s handwriting arose from the actions of the prosecutor.
C) Yes, because the process was not unduly suggestive since the prosecutor presented the teller with 10 customer signature cards.
D) Yes, because a lay witness may testify as to whether a document is in a person’s handwriting.

A

B) No, because the teller’s familiarity with the defendant’s handwriting arose from the actions of the prosecutor.

A lay witness with personal knowledge of a claimed author’s handwriting may testify as to whether a document is in that person’s handwriting. However, the lay witness must not have become familiar with the handwriting for the purpose of the current litigation.

How well did you know this?

1

Not at all

2

3

4

5

Perfectly

21

Q

A defendant was charged with fraud in connection with the sale of nutritional supplements. The prosecution alleged that the defendant verbally represented himself as a physician to convince elderly individuals to sign contracts authorizing the defendant to charge their credit cards monthly fees for deliveries of these supplements. In fact, the defendant had no medical training. The contracts did not identify the defendant as a physician, but the prosecution intended to introduce witness testimony that the defendant verbally represented himself as a physician. The sole issue in dispute is whether the defendant made such representations.

At trial, the prosecution introduced a photocopy of a contract between the defendant and one of the alleged victims in order to lay a foundation that the alleged victim bought supplements from the defendant. The defendant did not deny that the alleged victim bought supplements from him, but objected to the introduction of the contract on the ground that the prosecution was required to introduce an original contract under the best evidence rule. The court sustained the defendant’s objection.

Did the court err in making its ruling?

A) Yes, because the best evidence rule allows for the introduction of an original or duplicate document.
B) Yes, because the best evidence rule is not implicated in this case.
C) No, because the best evidence rule applies to a document that has a legal effect, such as a contract.
D) No, because a photocopy is not admissible when the absence of the original is not explained.

A

B) Yes, because the best evidence rule is not implicated in this case.

*Duplicates are admissible unless original’s authenticity is questioned or it would be unfair to admit them.

The best evidence rule requires that the original document or a reliable duplicate be produced to prove the contents of a writing. But this rule applies only when a witness is relying on the document when testifying or the contents of the document are at issue.

How well did you know this?

1

Not at all

2

3

4

5

Perfectly

22

Q

A boat owner initiated a products liability action against the manufacturer of the boat’s engine. The owner alleged that the engine manufacturer failed to warn the owner about the proper operation of a switch on the engine and that improper operation of the switch caused the owner’s injuries. The boat owner offered evidence that the manufacturer had begun including a written warning about the switch for all boats manufactured beginning in the year after the owner’s boat was manufactured. The owner had owned his boat for five years prior to his injury.

Is this evidence admissible?

A) No, because evidence of the manufacturer’s warning is inadmissible as a remedial measure.
B) No, because the remedial-measures exclusion is limited to negligence cases.
C) Yes, because the manufacturer began providing the warning before the boat owner’s accident.
D) Yes, because evidence of the need for a warning is not subject to the remedial-measures exclusion.

A

C) Yes, because the manufacturer began providing the warning before the boat owner’s accident.

Evidence of a remedial measure is inadmissible if it was undertaken by the defendant after the plaintiff was injured. A remedial measure undertaken before the plaintiff was injured is not subject to exclusion.

How well did you know this?

1

Not at all

2

3

4

5

Perfectly

23

Q

Two officers, a veteran and a novice, brought an experienced drug-sniffing dog on patrol one evening. They properly pulled over a driver on a bridge for erratic driving, and the veteran waited in the car to let the novice handle the stop. As the novice officer questioned the driver through the driver’s window, the dog circled the car and began barking and pawing desperately at the passenger-side door. The novice officer noticed that the driver had a package sitting on his passenger seat. When asked about the package, the driver responded that it was just meat from a butcher shop. As the novice officer stepped away from the window to write a ticket, the driver grabbed the package and threw it out of the passenger window and over the railing of the bridge. The dog immediately stopped barking. The veteran officer saw the whole event.

Assuming that the parties stipulate that the veteran is an expert in the training and reactions of drug-sniffing dogs, is the veteran officer’s testimony describing the dog’s reactions to the package admissible in a trial of the driver for possession of illegal drugs?

A) No, because the dog’s behavior was assertive conduct, making it hearsay not within any exception.
B) No, because the dog’s presence at the stop resulted in an improper search.
C) Yes, as evidence that the package contained illegal drugs.
D) Yes, because evidence generated by a machine or animal falls under an exception to the hearsay rule.

A

C) Yes, as evidence that the package contained illegal drugs.

The rule against hearsay bars the admission of an out-of-court statement made by a person—not a machine or animal—that is offered to prove the truth of the matter asserted therein.

Here, the prosecutor seeks to introduce the veteran officer’s testimony describing the dog’s reactions to the package. Although the dog’s behavior was assertive conduct, the veteran officer’s testimony does not implicate the hearsay rule because it relates to evidence generated by an animal

How well did you know this?

1

Not at all

2

3

4

5

Perfectly

24

Q

A witness to an armed robbery identified a suspect in a proper police lineup that was not attended by the suspect’s attorney. Charges were brought against the suspect, but the witness, a tourist from out of the country, had returned to her home country before the trial began. At trial, the prosecutor seeks to introduce the witness’s prior statement of identification into evidence. The defendant objects to the introduction of the evidence.

Should the court allow the prior statement of identification into evidence?

A) Yes, because it is admissible as nonhearsay.
B) Yes, because the witness is unavailable, so the statement of identification falls under a hearsay exception.
C) No, because the defendant’s attorney was not present at the identification.
D) No, because the witness is unavailable.

A

D) No, because the witness is unavailable.

A declarant’s prior statement that identifies a person as someone the declarant perceived earlier is nonhearsay if the declarant testifies and is subject to cross-examination about the statement.

Here, the witness’s statement identifies the defendant as someone the witness perceived earlier. But since the witness returned to her home country before trial, she is unavailable to testify and is not subject to cross-examination about her statement.

How well did you know this?

1

Not at all

2

3

4

5

Perfectly

25

Q

In a criminal trial for arson, a prosecution witness testifies under oath that she saw the defendant set fire to the victim’s home. The defendant’s attorney does not cross-examine the witness but seeks to introduce testimony that the witness gave at a deposition several months before the trial. At the deposition, the witness testified under oath that she did not see the defendant set fire to the victim’s home.

Should the court admit the deposition testimony?

A) The court should admit the witness’s deposition testimony for impeachment purposes but not as substantive evidence.
B) The court should admit the witness’s deposition testimony for impeachment purposes and as substantive evidence.
C) The court should not admit the witness’s deposition testimony because the defendant’s attorney did not allow the witness the chance to explain her inconsistent testimony.
D) The court should not admit the witness’s deposition testimony because it is hearsay.

A

B) The court should admit the witness’s deposition testimony for impeachment purposes and as substantive evidence.

A prior inconsistent statement is admissible nonhearsay if
(1) it was given under penalty of perjury at a trial, hearing, deposition, or other proceeding and
(2) the declarant testifies and is subject to cross-examination.

It can also be introduced extrinsically for impeachment purposes if the witness has the opportunity to explain or deny, and the adverse party can examine the witness about the statement.

Here, the witness’s prior inconsistent statement—that she did not see the defendant set fire to the victim’s home—was given under penalty of perjury at a deposition. And since the witness testified and was subject to cross-examination at trial, her deposition testimony is admissible nonhearsay and can be used as substantive evidence

How well did you know this?

1

Not at all

2

3

4

5

Perfectly

26

Q

A defendant is acquitted of murder. Subsequently, the family members of the victim bring a wrongful death action against the defendant. The defendant seeks to introduce a properly authenticated, certified copy of the final judgment to show that the defendant did not wrongfully kill the victim. The victim’s family members object to the introduction of the judgment.

May the defendant introduce the copy of the final judgment from his criminal case?

A) No, because a judgment in a criminal case is inadmissible in a subsequent civil action.
B) No, because the judgment is inadmissible hearsay.
C) Yes, because the copy of the judgment satisfies the original document rule.
D) Yes, because the level of proof in a civil action is less than that in the murder case.

A

B) No, because the judgment is inadmissible hearsay.

Hearsay: One exception exists for judgments of conviction. However, no such exception exists for judgments of acquittal

How well did you know this?

1

Not at all

2

3

4

5

Perfectly

27

Q

A defendant was charged with and tried for a crime. During the presentation of its case-in-chief, the prosecution introduced an inculpatory statement made by the defendant in an email regarding the commission of the crime. The defendant requested the immediate introduction of a subsequent related email sent by the defendant that contained an exculpatory statement. The defendant established that fairness requires that the two statements be considered at the same time.

Should the court honor the defendant’s request?

A) No, because the exculpatory statement was not made at the same time as the inculpatory statement.
B) No, because the statement the defendant seeks to introduce is exculpatory.
C) Yes, as a matter of judicial notice.
D) Yes, because fairness requires that the two statements be considered at the same time.

A

D) Yes, because fairness requires that the two statements be considered at the same time.

Rule of completeness = a party may introduce any part of a previously admitted writing or recorded statement, or any other writing or recorded statement, that in fairness should be considered at the same time

Note: The rule of completeness does not require that the responding writing or recorded statement be part of or made at the same time as the previously admitted statement.

How well did you know this?

1

Not at all

2

3

4

5

Perfectly

28

Q

A plaintiff sued a defendant under a disabilities discrimination statute, alleging that the defendant refused to hire the plaintiff because of her physical disability. The defendant has asserted that he refused to employ the plaintiff because he reasonably believed that she would be unable to perform the job. The defendant sought to testify that the plaintiff’s former employer advised him not to hire the plaintiff because she was unable to work productively for more than three hours each day.

Is the defendant’s testimony admissible?

A) No, because the defendant’s opinion of the plaintiff’s abilities is not based on personal knowledge.
B) No, because the former employer’s statement is hearsay not within any exception.
C) Yes, as evidence of the defendant’s reason for refusing to hire the plaintiff.
D) Yes, as evidence that the plaintiff would be unable to work longer than three hours each day.

A

C) Yes, as evidence of the defendant’s reason for refusing to hire the plaintiff.

Non-hearsay purpose: effect on the listener. Even if the former employer’s statements were not true, they had an effect on the defendant.

How well did you know this?

1

Not at all

2

3

4

5

Perfectly

29

Q

A plaintiff sued a defendant for libel after the defendant published an article on his website calling the plaintiff an adulterer. At trial, the defendant’s attorney called the plaintiff’s wife, who testified that on two separate occasions, the wife had found the plaintiff in a hotel with another woman. The plaintiff objected to the wife’s testimony.

Should the court sustain the plaintiff’s objection?

A) No, because character may be proven by specific instances of conduct in civil cases.
B) No, because the plaintiff’s infidelity is directly at issue in the trial.
C) Yes, because the wife’s testimony is barred by spousal privilege.
D) Yes, because the wife’s testimony is improper character evidence.

A

B) No, because the plaintiff’s infidelity is directly at issue in the trial.

Character evidence is admissible if a person’s character is an essential element of a civil claim, criminal charge, or asserted defense.

How well did you know this?

1

Not at all

2

3

4

5

Perfectly

30

Q

While driving through her neighborhood, a woman was involved in a car accident with her neighbor. The neighbor alleged that the woman failed to stop at a stop sign. The neighbor brought an action for negligence against the woman. At trial, the woman intends to call a witness to testify that the woman stops at the stop sign each time she encounters it. The witness is the woman’s coworker and frequently carpools with the woman to work, driving on a route with the same stop sign. However, the witness was not present for the accident with the neighbor.

Should the court admit the witness’s testimony?

A) No, because the testimony is improper character evidence.
B) No, because the witness did not observe the accident.
C) Yes, because it can be used to prove that the woman stopped at the stop sign on the day of the accident.
D) Yes, because the woman’s character for careful driving is an essential element of her defense.

A

C) Yes, because it can be used to prove that the woman stopped at the stop sign on the day of the accident.

Evidence of a person’s habit is admissible to prove that the person acted in accordance with that habit on a particular occasion.

Note: A witness who is familiar with the person—and his/her habit—may testify to prove the existence of that habit The witness need not have been present at the event in question to do so

How well did you know this?

1

Not at all

2

3

4

5

Perfectly

31

Q

The driver of a car was involved in an accident with a bicyclist. The bicyclist died as a result of his injuries, and the state has brought a vehicular manslaughter case against the driver, alleging that the driver caused the accident when he failed to stop at a red light. At trial, the driver calls his friend, who was not present at the scene of the accident, as a witness. The friend plans to testify that he knows that the driver is a safe driver, and that the driver could not have failed to stop at a red light on the day of the accident. The prosecution objects, seeking to exclude the friend’s testimony.

Which of the following is the best basis for the prosecution’s objection?

A) The friend is biased in favor of the driver.
B) The friend is not an expert on the issue of traffic accidents.
C) The friend lacks personal knowledge of the accident.
D) The friend’s testimony offers character evidence in a criminal case.

A

D) The friend’s testimony offers character evidence in a criminal case.

A lay witness may testify to any relevant matter of which he/she has personal knowledge.

This requires that the witness
(1) perceived the matter firsthand and
(2) have a present recollection of that observation

Note that the friend is likely biased in favor of the driver BUT OC can impeach the witness for bias at a later time, if the court allows the friend to testify

How well did you know this?

1

Not at all

2

3

4

5

Perfectly

32

Q

In a civil assault suit between a plaintiff and a defendant, a witness testified that the defendant had been with her on the night of the alleged assault, more than 200 miles away from where the assault was alleged to have occurred. To challenge the witness’s credibility, the plaintiff’s attorney sought to present evidence of the witness’s juvenile conviction for voluntary manslaughter five years ago. The defendant objected to the admission of this evidence.

Should the judge admit the evidence?

A) No.
B) No, but only if the judge concludes that the probative value of this evidence is outweighed by its prejudicial effect to the witness.
C) Yes, but only if the judge concludes that the probative value of this evidence outweighs its prejudicial effect to the witness.
D) Yes.

A

A) No.

Evidence of a juvenile conviction is never admissible in a civil case to attack a witness’s character for truthfulness.

How well did you know this?

1

Not at all

2

3

4

5

Perfectly

33

Q

A pedestrian was struck by a school bus while crossing a major intersection. The pedestrian brought suit against the bus company for negligence. At trial, the pedestrian testified that he always waits for the pedestrian-crossing signal before crossing major intersections. On cross-examination, the bus company asked the pedestrian about a ticket he had received for jaywalking at a major intersection three months before the accident in question.

Is the bus company’s question likely permissible?

A) No, because the probative value of the evidence is substantially outweighed by the danger of unfair prejudice.
B) No, because the receipt of a ticket for jaywalking is not probative of the truthfulness of the recipient of the ticket.
C) Yes, because the evidence contradicts the pedestrian’s testimony that he always waits for the pedestrian-crossing signal.
D) Yes, because the prior ticket demonstrates that the pedestrian did not wait for the crossing signal when the accident occurred.

A

C) Yes, because the evidence contradicts the pedestrian’s testimony that he always waits for the pedestrian-crossing signal.

A witness may be impeached by evidence that directly contradicts the witness’s testimony on a material issue. Both intrinsic and extrinsic evidence may be used for this purpose.

Note: The receipt of a ticket for jaywalking does not, in and of itself, relate to the truthfulness of the recipient. But in this instance, the ticket calls into question the pedestrian’s truthfulness as to his statement that he always waits for the pedestrian-crossing signal.

How well did you know this?

1

Not at all

2

3

4

5

Perfectly

34

Q

During the filming of an action movie, a stuntman was injured when he jumped out of a window and fell onto a negligently installed safety net. A cameraman, who had been hired to shoot behind-the-scenes footage, taped the stuntman falling onto the safety net and the net collapsing beneath him, causing him serious injury.

The stuntman has sued the movie producer for negligence and seeks to enter a digital copy of the cameraman’s video footage into evidence at trial. The stuntman also plans to call an expert witness to testify that, based upon the video, the stuntman’s jump out of the window followed all safety protocols and that the safety net could only collapse if it was not installed properly. The cameraman has already testified that, based on his personal knowledge, the video that the stuntman seeks to admit is an accurate digital copy of the video the cameraman recorded of the accident.

Assuming there is no genuine question as to the authenticity of the original video recording, is the digital copy of the video admissible?

A) No, because the digital copy violates the best evidence rule.
B) No, because the digital copy is not a reliable form of evidence as it can be easily manipulated.
C) Yes, because the cameraman has properly authenticated the digital copy.
D) Yes, because the expert witness can guide the jury through the video and explain how and why the stuntman was injured.

A

C) Yes, because the cameraman has properly authenticated the digital copy.

Here, there is no question about the authenticity of the original recording and no indication that circ*mstances make it unfair to admit a copy. Therefore, admission of the copy does not violate the best evidence rule.

Note: The expert witness can likely guide the jury through the video and explain how and why the stuntman was injured. But this has no bearing on the admissibility of the recording.

How well did you know this?

1

Not at all

2

3

4

5

Perfectly

35

Q

A defendant was charged with theft of merchandise from a store. On the witness stand, the defendant admitted to taking the merchandise on the day in question but contended that she lacked the intent to do so. A rebuttal witness testified that she was standing outside the store after purchasing something, and she saw the defendant outside the store furtively removing the merchandise from her coat. When asked about her recollection of the date, the witness testified that she knew that it was the day in question because that date was on her receipt. The defendant objected, asserting that the prosecution must produce the receipt.

How is the court likely to rule on this objection?

A) Overrule the objection, because the date is a collateral issue.
B) Overrule the objection, because the date is irrelevant.
C) Sustain the objection, because the receipt is the most reliable evidence of the date.
D) Sustain the objection, because the witness’s knowledge of the date is based on the receipt.

A

A) Overrule the objection, because the date is a collateral issue.

Under the best evidence rule, an original or reliable duplicate is not required, and other evidence of content is admissible, when the contents go toward a collateral issue.

Note: Here, the witness relied on the date on the receipt when testifying about the date of the theft. But the date of the theft is collateral (i.e., undisputed) because the defendant has already admitted to taking the merchandise on the day in question. This means that the prosecution need not produce the receipt (or a reliable duplicate) for the witness to testify about it.

How well did you know this?

1

Not at all

2

3

4

5

Perfectly

36

Q

A plaintiff brought a defamation action against his friend, an internet blogger, for the publication of defamatory accusations against the plaintiff that the friend allegedly published on his internet blog. The plaintiff testified that because he had always been a daily reader of the friend’s writing, he read the defamatory remarks on June 10, the same day they were published. When the plaintiff’s attorney asked the plaintiff how he remembered the date, the plaintiff answered, “When I called his house to demand that he take down the post, his girlfriend answered the phone and said that he was out seeing a movie that had come out that day.” The friend’s attorney objected and moved to strike the testimony.

Should the court strike the plaintiff’s testimony about the girlfriend’s statement on the phone?

A) No, because the court may take judicial notice of the movie release date.
B) No, because the statement is not being offered for its truth.
C) Yes, because a court may refuse to admit evidence related to a collateral issue.
D) Yes, because the plaintiff has not established that the girlfriend is unavailable to testify.

A

B) No, because the statement is not being offered for its truth.

An out-of-court statement implicates the rule against hearsay only when it is offered to prove the truth of the matter asserted therein. Therefore, a statement offered for some other purpose is not barred by this rule.

Here, the plaintiff is offering the girlfriend’s statement to show how he was able to recall the date on which the accusations were published—not to prove that the friend in fact went to the movies on that day. Since the statement is not being offered for its truth, it is not barred by the hearsay rule.

How well did you know this?

1

Not at all

2

3

4

5

Perfectly

37

Q

A defendant is on trial for embezzling $50,000 from his former employer. The prosecution wishes to offer into evidence an anonymous letter, in its entirety, that was received by the defendant’s former supervisor. The supervisor testified that the letter was written in the defendant’s handwriting, which the supervisor knew from their years of working together. The letter reads, “I am consumed by guilt for what I have done. Here is half the money I took from you, and if you promise not to prosecute, I will send you the rest later this year. If you accept this arrangement, please post a personal ad in the local paper using the phrase ‘All is forgiven.’” The note was accompanied by $25,000 in cash. The defense objects to the admission of the letter.

Is the letter admissible?

A) No, because public policy calls for the exclusion of statements made in a negotiation to settle a claim.
B) No, because the letter has not been properly authenticated.
C) Yes, because the letter contains statements by an opposing party to the current litigation.
D) Yes, because statements of the declarant’s present intent, motive, or plan are excepted from hearsay.

A

C) Yes, because the letter contains statements by an opposing party to the current litigation.

Note: The prosecution authenticated the letter when it had the supervisor, a lay witness with personal knowledge of the defendant’s handwriting, testify that the letter was written by the defendant.

How well did you know this?

1

Not at all

2

3

4

5

Perfectly

38

Q

A woman sued her neighbor for conversion, alleging that the neighbor and his nephew took the woman’s vintage automobile from her garage and sold it on the black market. Prior to trial, the nephew was in a car accident in which his leg was badly broken. The nephew was rushed to the hospital for emergency surgery. On his way into surgery, the nephew told a nurse that he and the neighbor had taken and sold the woman’s automobile. The nephew later died in surgery due to unforeseen complications.

Which of the following hearsay exceptions will allow admission of the nurse’s testimony as to the nephew’s statement?

A) Dying declaration.
B) Excited utterance.
C) Statement against interest.
D) Statement of then-existing state of mind.

A

C) Statement against interest.

The statement against interest exception applies to hearsay statements
(1) that are contrary to an unavailable declarant’s proprietary or pecuniary interest;
(2) tend to invalidate the declarant’s claim against someone else, or
(3) expose the declarant to civil or criminal liability.

Note: The “dying declaration” exception applies to statements that (1) were made while an unavailable declarant believed his/her death was imminent and (2) concerned the circ*mstances of that impending death. But here, there is no indication that the nephew believed his death was imminent, and his statement did not pertain to the circ*mstances of his death.

How well did you know this?

1

Not at all

2

3

4

5

Perfectly

39

Q

A plaintiff filed suit against a supermarket for injuries that he sustained when he slipped on a piece of lettuce in the supermarket’s produce aisle. A supermarket employee who witnessed the plaintiff’s fall immediately prepared a written summary of the events that had occurred in order to alert the supermarket’s management. At trial, the supermarket’s lawyer called the employee as a witness. The employee testified that she could not recall the events, even after looking through her written summary while on the witness stand. The supermarket’s lawyer then asked her to read her summary to the jury. The plaintiff objected to the testimony.

May the court admit the testimony at this time over the plaintiff’s objection?

A) No, because it is hearsay not within any exception.
B) No, because such testimony may only be offered into evidence by an adverse party.
C) Yes, because the employee is on the witness stand and can be cross-examined.
D) Yes, because the employee is unable to remember the actual events even after referencing her summary.

A

D) Yes, because the employee is unable to remember the actual events even after referencing her summary.

Recorded recollection: allows a record to be read into evidence if it (
1) concerns a matter that a witness once knew but cannot recall at trial; (2) was made or adopted by the witness when the matter was fresh in his/her mind, and
(3) accurately reflects the witness’s personal knowledge at the time it was made.

Note: The employee is on the witness stand and can be cross-examined. But this is not a basis to admit a witness’s hearsay statement. Instead, the statement must fall under a hearsay exclusion or exception—e.g., the recorded recollection exception

How well did you know this?

1

Not at all

2

3

4

5

Perfectly

40

Q

A defendant is on trial for burglary. The jurisdiction has a statute that a charge of burglary may be elevated to aggravated burglary if the crime is committed with a firearm. An eyewitness, who is the burglary victim’s neighbor, called the police on the night in question and reported that she saw the defendant climb out of the victim’s window and that he was carrying a gun. The eyewitness wrote a detailed description of the incident and the weapon in her diary soon after she called the police.

During the trial, the prosecutor put the eyewitness on the stand, but she could no longer recall whether the defendant was holding anything. The prosecutor asked her to read the diary entry to herself to see if it refreshed her memory. When the eyewitness admitted that it did not, the prosecutor sought to have the eyewitness read the diary entry to the jury and to introduce the diary entry as an exhibit. The defendant objects to both.

How should the court rule?

A) The court should sustain both objections.
B) The court should sustain the objection as to the eyewitness’s reading of the diary entry but overrule the objection as to entering it as an exhibit.
C) The court should overrule the objection as to the eyewitness reading the diary entry to the jury but sustain the objection as to entering it as an exhibit.
D) The court should overrule both objections.

A

C) The court should overrule the objection as to the eyewitness reading the diary entry to the jury but sustain the objection as to entering it as an exhibit.

Recorded recollection: only an adverse party may introduce the record

Here, the eyewitness’s memory was not refreshed by having her read the diary entry to herself. And since the diary entry satisfies the criteria for admission under the recorded recollections hearsay exception, the court should allow the eyewitness to read the diary entry into evidence

How well did you know this?

1

Not at all

2

3

4

5

Perfectly

41

Q

A defendant is on trial for armed robbery and felony murder. The prosecution seeks to admit testimony by the first witness to arrive at the scene of the crime. The witness discovered the victim just before he died of a gunshot wound, and the victim identified the defendant as his assailant only moments before he died. The defense asserts that the victim was too delirious from blood loss to know that he was dying and hopes to present a statement from the victim’s widow to support this assertion. However, both parties agree that the widow’s statement is privileged under federal law.

In what manner should the court determine whether the victim’s statement is a dying declaration?

A) Allow the prosecution to admit the testimony only if the judge determines that the witness is credible.
B) Allow only the unprivileged evidence from both sides at the jury trial so the jury may decide whether the victim believed he was dying.
C) Consider all of the evidence from both sides outside the presence of the jury.
D) Consider only the unprivileged evidence from both sides outside the presence of the jury.

A

D) Consider only the unprivileged evidence from both sides outside the presence of the jury.

The court must decide preliminary questions of fact related to whether evidence is admissible, a privilege exists, or a witness is qualified.

Any hearing on these matters must be conducted outside the jury’s presence if
(1) the matter involves the admissibility of a confession;
(2) a defendant in a criminal case is a witness and so requests: OR
(3) justice so requires.

The court may only consider unprivileged evidence when making this determination, so the widow’s privileged statement may not be considered. And since the victim’s statement identifying the defendant as the assailant would clearly prejudice the defendant, the hearing should be conducted outside the jury’s presence.

Note: the court must decide the preliminary question of whether the victim knew that he was dying before ruling on the admissibility of the victim’s statement as a dying declaration.

How well did you know this?

1

Not at all

2

3

4

5

Perfectly

42

Q

A woman is on trial for a burglary that took place at about 6:00 p.m. on November 1. A surveillance video from a local gas station shows that the woman visited the gas station at 7:00 p.m. on November 1. The woman alleges as an alibi that she was actually at the gas station at 6:00 p.m. She argues that the gas station failed to change the time display on its camera to reflect the end of Daylight Savings Time on the morning of November 1. The judge, on his own initiative and after first giving the prosecutor the opportunity to object, took judicial notice of the fact that Daylight Savings Time did end in the year in question on November 1. The judge instructed the jury that it may or may not accept any judicially noticed fact as conclusive.

Were the judge’s actions with regard to judicial notice of this fact proper?

A) No, because the court should have instructed the jury that it is required to accept the noticed fact as conclusive.
B) No, because the fact is not one that is generally known within the territorial jurisdiction of the trial court.
C) Yes, because the court gave the prosecution an opportunity to be heard on the propriety of taking judicial notice before doing so.
D) Yes, because the court may take judicial notice on its own initiative.

A

D) Yes, because the court may take judicial notice on its own initiative.

A court may take judicial notice of any adjudicative fact that is not subject to reasonable dispute because it
(1) is generally known within the territorial jurisdiction of the trial court or
(2) can be accurately and readily determined from sources whose accuracy cannot reasonably be questioned.

But this fact can be accurately and readily determined from sources whose accuracy cannot be reasonably questioned (e.g., calendar, almanac), so the court may take judicial notice of this fact on its own initiative.

Note: a judge is not required to provide this opportunity before taking judicial notice of an adjudicative fact.

How well did you know this?

1

Not at all

2

3

4

5

Perfectly

43

Q

A female and a male candidate were both competing for a junior coaching position with a college football team. Although the female candidate was more qualified and experienced, the head coach chose the much less competent male candidate for the coaching position. The female candidate subsequently sued the head coach for employment discrimination. At trial, the female candidate’s lawyer sought to introduce testimony by a football player on the team claiming that the head coach had told him that “women don’t belong on the football field unless they are wearing a cheerleading uniform.”

Is the head coach’s statement admissible?

A) No, because the probative value of the statement is substantially outweighed by its prejudicial effect.
B) No, because the statement is hearsay that does not fall within an exception.
C) Yes, because it is relevant to the claim of employment discrimination against the head coach.
D) Yes, because the statement falls within the state of mind exception to the hearsay rule.

A

C) Yes, because it is relevant to the claim of employment discrimination against the head coach.

Non-hearsay: illustrate the declarant’s world view or belief system.

Note: The then-existing state of mind exception to the hearsay rule allows statements of a declarant’s then-existing mental state—e.g., motive, intent, or plan—to be admitted as substantive proof that the declarant later acted in accordance with that mental state. But here, the coach’s statement is not hearsay, so no exception is needed. Additionally, the statement is not one of motive, intent, or plan.

How well did you know this?

1

Not at all

2

3

4

5

Perfectly

44

Q

A defendant is on trial for the crime of menacing due to allegedly making threatening phone calls to a woman living in his apartment building. The prosecution called a female witness who lived in the defendant’s prior apartment building to testify that she also received a number of unidentified but identical threatening phone calls while the defendant lived in her building. The defense objected to the testimony on the ground of relevance. The prosecution responded by explaining that it plans to introduce further evidence establishing that the calls received by this witness were made by the defendant.

Is the witness’s testimony admissible?

A) No, because calls by an unidentified caller are not relevant to the case.
B) No, because the witness’s statement cannot be admitted prior to the production of evidence establishing that the defendant made the calls to the witness.
C) Yes, on the condition that evidence is introduced later that would permit the jury to reasonably find by a preponderance of the evidence that the caller was the defendant.
D) Yes, on the condition that the court finds by a preponderance of the evidence that the caller was the defendant.

A

C) Yes, on the condition that evidence is introduced later that would permit the jury to reasonably find by a preponderance of the evidence that the caller was the defendant.

Conditional relevance: When the relevance of evidence depends on whether a fact exists, proof must be introduced to allow the court to determine whether the jury could reasonably find the conditional fact by a preponderance of the evidence. However, the court may admit the proposed evidence on the condition that such proof be introduced later.

How well did you know this?

1

Not at all

2

3

4

5

Perfectly

45

Q

A plaintiff manufacturer brought an action for breach of contract against a defendant retailer for the amount due under a contract for a shipment of widgets. The contract, which was admitted into evidence, indicated that the plaintiff was entitled to payment of the contract price once the widgets were delivered to a commercial carrier. The plaintiff called a witness who has worked in the plaintiff’s shipping warehouse for many years. The witness testified that the shipment of widgets ordered by the defendant was delivered to the commercial carrier with instructions to deliver the widgets to the defendant’s shipping address. The receipt for this delivery to the commercial carrier was entered into evidence.

When the defendant’s attorney asked the witness how he knew the defendant’s shipping address, the witness stated, “We keep all of our customers’ addresses in our shipping records, but I know that one by heart because they have been a regular customer for years. They order a lot of widgets, and they always call to make sure I used the right address.” The defendant’s attorney then objected to the witness’s testimony about the defendant’s shipping address and asked that the testimony be stricken from the record on the ground that the shipping records had never been shown to the defense or offered as evidence.

Should the court sustain the defendant’s objection to the witness’s testimony?

A) No, because the witness can testify about the plaintiff’s regularly conducted business with the defendant.
B) No, because the witness had personal knowledge of the shipping address used to send the shipment of widgets.
C) Yes, because the defense has not been given the opportunity to examine the plaintiff’s shipping records.
D) Yes, because the plaintiff must offer its original shipping records to establish the defendant’s shipping address.

A

B) No, because the witness had personal knowledge of the shipping address used to send the shipment of widgets.

A witness may testify to any relevant fact about which he/she has personal knowledge—i.e., firsthand experience or observation—even when other evidence may contain the same information.

Note: The mere fact that the witness can testify about the plaintiff’s regularly conducted business with the defendant is not sufficient for the witness to testify about the defendant’s shipping address. Instead, the witness can testify about the shipping address because he has personal knowledge of it

How well did you know this?

1

Not at all

2

3

4

5

Perfectly

46

Q

A car struck a truck at an intersection. The driver of the truck sued the driver of the car, claiming that the car driver ran a red light. At trial, the only witness to the accident testified that he clearly remembered that the car’s traffic light had been red and that the car ran the light. However, in the investigating officer’s report, which was made hours after the accident, the witness is quoted saying, “I saw the whole thing. The car had the green light.” The car driver did not cross-examine the witness, and the witness was dismissed and left the jurisdiction.

After the truck driver presented his case, the car driver moved to introduce the witness’s statement from the investigating officer’s report solely to impeach the witness’s testimony. The truck driver objected.

How should the court rule?

A) Overrule the objection, and admit the statement as impeachment evidence only.
B) Overrule the objection, and admit the statement as substantive evidence that the car driver did not run the red light.
C) Sustain the objection, because extrinsic evidence may not be used to impeach a witness under these circ*mstances.
D) Sustain the objection, because the statement is inadmissible hearsay.

A

C) Sustain the objection, because extrinsic evidence may not be used to impeach a witness under these circ*mstances.

Note: Extrinsic evidence of a witness’s prior inconsistent statement is admissible only if the impeached witness has the opportunity to explain or deny—and the adverse party can examine the witness about—the statement (or if justice so requires).

How well did you know this?

1

Not at all

2

3

4

5

Perfectly

47

Q

The defendant in a civil fraud case plans to testify regarding the sale of land at issue in the case. The plaintiff seeks, for the sole purpose of impeaching the defendant’s character for truthfulness, to introduce evidence of the defendant’s recent conviction for felony assault. The defendant has filed a motion to exclude evidence of the conviction, and the judge has scheduled a hearing on the defendant’s motion.

Which of the following accurately describes each party’s burden regarding introduction of the conviction?

A) The defendant must show that the probative value of the conviction is outweighed by the prejudicial effect in order to prevail.
B) The defendant must show that the probative value of the conviction is substantially outweighed by the prejudicial effect in order to prevail.
C) The plaintiff must show that the probative value of the conviction outweighs its prejudicial effect in order to prevail.
D) The plaintiff must show that the probative value of the conviction substantially outweighs its prejudicial effect in order to prevail.

A

B) The defendant must show that the probative value of the conviction is substantially outweighed by the prejudicial effect in order to prevail.

Convictions for felonies not involving dishonesty that are no more than 10 years old are admissible against a civil witness unless the party opposing the introduction of the conviction shows that its probative value is substantially outweighed by its prejudicial effect.

Note: The burden is on the defendant—as the party opposing the admission of the conviction—to show that the probative value of the felony assault is substantially outweighed by its prejudicial effect.

48

Q

A landowner sued a defendant for trespass and destruction of property. The defendant called a witness to testify that at the time of the alleged trespass, the defendant was with the witness miles away from the landowner’s property. On cross-examination, the landowner asked the witness if he was the defendant’s best friend.

Is this question proper?

A) No, because it exceeds the scope of the defendant’s direct examination of the witness.
B) No, because it is not relevant to the issue of whether the defendant trespassed on and destroyed the landowner’s property.
C) Yes, because character evidence is admissible in a civil action to prove conformity.
D) Yes, because it impeaches the witness’s testimony.

A

D) Yes, because it impeaches the witness’s testimony.

Impeachment = bias

49

Q

A man sued his neighbor for assault. The man testified that on the three nights that his neighbor had threatened to hurt him, his wife had recorded the events in her diary. However, the man could not remember the dates of the events or the reasons that the neighbor made the threats. The man’s attorney permitted the man to examine his wife’s diary while on the stand. After a brief examination of the diary, the man then testified to the exact dates of the threats and the reasons that the neighbor made the threats. The neighbor seeks to introduce the relevant portions of the diary into evidence to prove that the man was lying.

Can the diary be entered into evidence?

A) No, because it can only be used to refresh the man’s recollection while testifying.
B) No, because it is hearsay not subject to an exception.
C) Yes, because a writing used to refresh a witness’s recollection may always be admitted for substantive purposes.
D) Yes, because the neighbor is entitled to enter into evidence any portion of the diary relevant to the assault claim to impeach the man.

A

D) Yes, because the neighbor is entitled to enter into evidence any portion of the diary relevant to the assault claim to impeach the man.

Once a witness has used a writing to refresh his/her recollection, the adverse party is entitled to (
1) have the writing produced for inspection,
(2) cross-examine the witness about the writing, and
(3) introduce into evidence any portion of the writing that relates to the witness’s testimony.

Here, the man examined his wife’s diary on the stand for the purpose of refreshing his recollection. As a result, the neighbor is entitled to introduce any relevant portions of the diary into evidence to prove that the man was lying—i.e., to impeach the man. Accordingly, the diary can be entered into evidence.

50

Q

A plaintiff hopes to introduce an original x-ray image of his broken arm at trial to help prove the injury element of his negligence claim against the defendant. The plaintiff called a doctor to lay the foundation in order to enter the image into evidence. The doctor testified that the image was an accurate depiction of the plaintiff’s arm taken the day after the alleged injury. The doctor also testified that he had taken the x-ray image himself.

If the above testimony is the only testimony presented about the x-ray image, is the image admissible?

A) No, because the best evidence rule bars the image’s admission.
B) No, because the image has not been properly authenticated.
C) Yes, because the doctor authenticated the image.
D) Yes, because the image is self-authenticating.

A

B) No, because the image has not been properly authenticated.

Here, the doctor merely testified that the x-ray image was an accurate depiction of the plaintiff’s arm taken the day after the alleged injury and that he had taken the image. But since the x-ray image is a physical representation of something that cannot otherwise be seen, this testimony was insufficient to properly authenticate the image

(1) the process for creating the evidence was accurate, (2) the machine that produced the evidence was working properly, and
(3) the operator of the machine was qualified to operate it.

51

Q

A client sued a lawyer for malpractice for recording the wrong property deed with the county register of deeds. At trial, the attorney representing the client seeks to introduce into evidence a certified copy of the deed that was recorded with the register, who is the proper officer with whom to record deeds pursuant to state law. The defendant-lawyer objected to the introduction of the evidence.

Should the judge admit the evidence?

A) No, as it is hearsay not within any exception.
B) No, because the original should be presented since the contents are at issue.
C) Yes, if a representative of the register of deeds testifies to lay a foundation for admissibility.
D) Yes.

A

D) Yes.

Copies of documents recorded and filed in a public office as authorized by law are self-authenticating if certified as correct.

The best evidence rule generally requires that an original document be produced when the contents of the document are at issue. This rule is not implicated here because the contents of the deed are not at issue. What is at issue is whether the defendant-lawyer filed the correct deed with the register

52

Q

An animal rights activist is on trial for the burglary of a pharmaceutical lab. The prosecution’s theory of the case is that the activist broke into the lab using a maintenance access tunnel connecting the lab to a neighboring building. The tunnel was difficult to locate by anyone unfamiliar with its existence. The prosecution hopes to call an intern who works for the architect of the lab to testify that the activist visited the architect’s office on several occasions to interview the architect, who is now out of the country, for a story. The prosecution further hopes to have the intern testify that the activist had ample time to inspect a three-dimensional model of the lab that sat in the architect’s office and clearly depicted the access tunnel. The model has since been recycled as part of the architect’s regular business practices and cannot be retrieved to be entered as evidence. The intern has no knowledge of the lab or access tunnel independent of the model. Although the intern was included on the prosecution’s witness list, the prosecution did not provide advance notice of the intern’s testimony.

Can the intern properly testify as to the appearance of the model?

A) No, because the prosecution did not establish that no other reproduction of the model exists.
B) No, because the prosecution did not give advance notice of this oral testimony.
C) Yes, because the best evidence rule does not apply.
D) Yes, because the model was not destroyed in bad faith.

A

C) Yes, because the best evidence rule does not apply.

Real or physical evidence is not subject to best evidence rule, only recordings, writings, or photographs

53

Q

In a probate proceeding, the beneficiary of a purported holographic will sought to introduce a photocopy of that will as evidence of the contents of the missing original holographic will. An heir of the testator opposed the introduction of this photocopy into evidence. Contending that the testator was not of sound mind when the original will was written, the heir demanded that the beneficiary produce the original will. The applicable jurisdiction recognizes the validity of a holographic will and has adopted a dead man’s statute.

Is the photocopy of the will admissible without an explanation of the unavailability of the original?

A) No, because the will is a document that has legal effect.
B) No, because there is a question as to the authenticity of the will.
C) Yes, because the dead man’s statute permits the introduction of the photocopy.
D) Yes, because the photocopy is a duplicate of the original.

A

D) Yes, because the photocopy is a duplicate of the original.

Best evidence rule: a duplicate is admissible to the same extent as the original unless (1) a genuine question is raised about the original’s authenticity or (2) the circ*mstances make it unfair to admit the duplicate.

54

Q

A defendant was charged with the murder of her own child. Two months prior to trial, the prosecutor interviewed a family friend who had known the defendant her whole life. During the interview, the prosecutor asked if the defendant had ever committed violent acts against animals as a child. The friend responded with a nod of his head up and down. He then stated, “I often saw bruises on [the defendant’s] child but figured they were from playing.”

During the defendant’s trial, the prosecution sought to introduce into evidence the statement from the friend, as well as the fact that the friend nodded in response to the prosecutor’s question, to demonstrate that the defendant had a violent history against animals and that the defendant’s child often had bruises. The friend died just prior to the start of the trial. The defendant has objected to this evidence on hearsay grounds.

How should the court rule on the admissibility of the aforementioned evidence?

A) Admit the evidence of the nod only as nonhearsay.
B) Admit the evidence of the nod and the statement as nonhearsay.
C) Admit the evidence of the nod and the statement as hearsay falling under an exception.
D) Refuse the admission of both the nod and the statement.

A

D) Refuse the admission of both the nod and the statement.

Here, the prosecution seeks to admit the family friend’s nonverbal affirmative head nod as substantive proof that the defendant committed violent acts against animals as a child. Therefore, the head nod is hearsay and is inadmissible unless excluded or excepted from the hearsay rule. The friend is unavailable as a witness since he died prior to trial, which broadens the possible hearsay exceptions that could apply to his assertion. But since no exclusion or exception applies, the court should refuse its admission

55

Q

The owner of an electronics store brought a civil suit for the value of stolen electronics against one of his former employees, who had previously been convicted in a criminal court for the theft of the same goods. During the civil trial, the plaintiff-owner called a witness whom he hoped would testify that she saw the defendant in possession of the stolen goods the day after the electronics store was robbed. The witness, however, testified that she did not see the defendant in possession of the goods and that she was actually out of town the day after the robbery. The plaintiff seeks to introduce the witness’s testimony from the criminal case, in which she testified that she saw the defendant in possession of the goods the next day. The defendant objects to the introduction of the testimony.

Should the court allow the testimony into evidence?

A) Yes, for impeachment only.
B) Yes, as substantive evidence only.
C) Yes, both for impeachment and as substantive evidence.
D) No, not for any reason.

A

C) Yes, both for impeachment and as substantive evidence.

Here, the witness’s prior inconsistent statement was given under penalty of perjury at the defendant’s criminal trial.

And since the witness testified and was subject to cross-examination at the civil trial, her prior inconsistent testimony is admissible non-hearsay and can be used as substantive evidence

56

Q

A man witnessed a hit-and-run accident in which a pickup truck struck and killed a child. The next day, the witness gave police a signed, handwritten statement with a description of the truck, including the make and model, as well as a description of the driver of the truck. After several months, the authorities identified and charged a man who fit the witness’s description, although the truck was never located. In the interim, however, the witness had suffered a brain injury that left his memory spotty.

At the man’s trial, the prosecutor called the witness to testify about the accident but the witness had difficulty remembering key details. The prosecutor first showed the witness a photo of a vehicle of the same make and model as the truck in an attempt to refresh the witness’s memory. The defense objected but was overruled by the judge. When this failed to refresh the witness’s memory, the prosecutor showed the witness his earlier handwritten and signed statement. Again, the defense objected and was overruled. The witness testified that he still could not remember the details of the accident, but that he recognized his handwriting on the statement and that the statement accurately reflected what he witnessed at the time. The prosecutor then moved to introduce the photo and statement into evidence as exhibits, and the defense again objected.

How should the judge rule?

A) The judge should admit both the photo and the statement into evidence as exhibits.
B) The judge should admit the photo, but not the statement, into evidence as an exhibit.
C) The judge should admit the statement, but not the photo, into evidence as an exhibit.
D) The judge should not admit the photo or the statement into evidence as exhibits.

A

D) The judge should not admit the photo or the statement into evidence as exhibits.

A witness’s memory can be refreshed by allowing the witness to review any item before or while testifying. The item need not be admissible to be used for this purpose. But if a party seeks to admit the item, then the item must satisfy all evidentiary rules—e.g., relevance, authentication, hearsay.

Here, the prosecutor seeks to admit a photograph of a vehicle of the same make and model as the truck allegedly involved in the hit and run. To be admissible, the photograph must be relevant—i.e., have any tendency to make a material fact more or less probable. But since the photo merely depicts a truck similar to the man’s alleged truck, it is not relevant. Therefore, the court should not admit the photo

57

Q

A defendant, his cousin, and a friend planned to rob a convenience store. The friend watched the defendant and his cousin put on ski masks and enter the convenience store. Moments later, the friend heard a gunshot and watched as the defendant fled the convenience store. The cousin stumbled out of the convenience store, pointed at his bleeding foot, and told the friend, “He shot me! The moron dropped his gun and shot me. I’m going to kill that idiot!” The cousin ran after the defendant. The next day, the cousin was found dead from a bullet wound to the chest. Police later charged the defendant with murder.

At trial, the defendant claimed self-defense and seeks to introduce the cousin’s statement to the friend as evidence that the cousin was the initial aggressor. The prosecution objects.

What is the defendant’s best argument for the court to find that the statement is admissible?

A) The declarant is unavailable as a witness.
B) The statement indicated the cousin’s present intent.
C) The statement was a dying declaration.
D) The statement was made by a coconspirator.

A

B) The statement indicated the cousin’s present intent.

Statements of the declarant’s then-existing state of mind—e.g., statements of motive, present intent, or plan—are excepted from hearsay. This is true regardless of whether the declarant is available as a witness

58

Q

In a civil trial for professional malpractice, the plaintiff sought to show that the defendant, an engineer, had designed the plaintiff’s flour mill with inadequate power. The plaintiff called an expert witness who based his testimony solely on his own professional experience but also asserted, when asked, that the book Smith on Milling Systems was a reliable treatise in the field and consistent with his views. On cross-examination, the defendant asked the witness whether he and Smith were ever wrong. The witness answered, “Nobody’s perfect.” The defendant asked no further questions. The defendant later called his own expert witness and asked, “Do you accept the Smith book as reliable?” The witness said, “It once was, but it is now badly out of date.” The plaintiff requested that the jury be allowed to examine the book and judge for itself the book’s reliability.

Should the court allow the jury to examine the book?

(A) No, because the jury may consider only passages read to it by counsel or by a witness.
(B) No, because the plaintiff’s expert did not rely on the treatise in his testimony but on his own experience.
(C) Yes, because an expert has testified that the treatise is reliable.
(D) Yes, because the jury is the judge of the weight and credibility to be accorded both written and oral evidence.

A

(A) No, because the jury may consider only passages read to it by counsel or by a witness.

Learned treatise rule: if the court finds a publication to be a reliable authority, then “statements” from it may be read into evidence, but the publication may not be received as an exhibit.

Note: There is a concern that if juries were allowed unrestricted access to the whole publication, they might rely on parts of the publication that are not germane to the case

59

Q

At a defendant’s trial for a gang-related murder, the prosecution introduced, as former testimony, a statement by a gang member who testified against the defendant at a preliminary hearing and has now invoked his privilege against self- incrimination.

If the defendant now seeks to impeach the credibility of the gang member, which of the following is the court most likely to admit?

(A) Evidence that the gang member had three misdemeanor convictions for assault.
(B) Testimony by a psychologist that persons with the gang member’s background have a tendency to fabricate.
(C) Testimony by a witness that, at the time the gang member testified, the gang member was challenging the defendant’s leadership role in the gang.
(D) Testimony by a witness that the gang member is a cocaine dealer.

A

C) Testimony by a witness that, at the time the gang member testified, the gang member was challenging the defendant’s leadership role in the gang.

This is evidence of “bias.” It shows that the gang member had a motive to implicate the defendant falsely, because by doing so he would remove the defendant from the position that he wanted to have,

Note: Expert testimony on credibility is usually found inadmissible because credibility issues are for the jury, not for the imprimatur of an expert.

60

Q

A defendant was charged with aggravated assault. At trial, the victim testified that the defendant beat her savagely, but she was not asked about anything said during the incident. The prosecutor then called a witness to testify that when the beating stopped, the victim screamed: “I’m dying—don’t let [the defendant] get away with it!”

Is the testimony of the witness concerning the victim’s statement admissible?

A) No, because it is hearsay not within any exception.
(B) No, because the victim was not asked about the statement.
(C) Yes, as a statement under belief of imminent death, even though the victim did not die.
(D) Yes, as an excited utterance.

A

(D) Yes, as an excited utterance.

In this case, the assault was a startling event, and the victim made the statement immediately after the beating, trying to identify the perpetrator

Note: NOT a dying declaration for two reasons
1) the declarant has to be unavailable, as the dying declaration is one of the “unavailability-dependent” exceptions of Rule 804. Here, the victim testified and so obviously is not unavailable.
2) dying declaration is admissible only in homicide prosecutions and civil cases.

61

Q

A woman who is a computer expert decided to dedicate herself to exposing persons who trafficked in child p*rnography. She posted a number of sexually oriented photographs on her website. The file for each photograph contained an embedded Trojan horse program (a program that would allow the woman to enter the computer of anyone who downloaded the photograph). A man downloaded one of those photographs onto his personal computer. Using the embedded program, the woman entered the man’s computer and found a file containing a p*rnographic photograph of a child. She copied the file and turned it over to a federal law enforcement agency. A federal agent told her that a successful prosecution would require more than one photograph and offered her a monetary reward for additional photographs leading to the man’s conviction. The woman entered the man’s computer again, and this time she found hundreds of child p*rnography photographs, which she turned over to the federal agency.

The man was charged with multiple counts of violating federal child p*rnography statutes. He has moved to suppress the photographs that the woman discovered on his computer. The motion is based on both the Fourth Amendment and a federal statute forbidding interception of electronic communication without permission. The parties have stipulated that the woman’s conduct in downloading photographs from the man’s computer violated the interception statute.

How should the court rule on the defendant’s motion to suppress?

A) Deny the motion as to all photographs.
(B) Grant the motion as to all photographs, because the woman acted without probable cause.
(C) Grant the motion as to all photographs, because the woman violated the federal interception statute.
(D) Grant the motion only as to the second set of photographs.

A

(D) Grant the motion only as to the second set of photographs.

Because authorities encouraged and offered to reward the woman for the second computer search, the woman was acting as a government agent with regard to that search, which violated the Fourth Amendment because it was conducted without a warrant.

62

Q

In a personal injury case, the plaintiff sued a retail store for injuries she sustained from a fall in the store. The plaintiff alleged that the store had negligently allowed its entryway to become slippery from snow tracked in from the sidewalk. Before the lawsuit was filed, when the plaintiff first threatened to sue, the store’s manager said, “I know that there was slush on that marble entryway, but I think your four-inch-high heels were the real cause of your fall. So let’s agree that we’ll pay your medical bills, and you release us from any claims you might have.” The plaintiff refused the offer. At trial, the plaintiff seeks to testify to the manager’s statement that “there was slush on that marble entryway.”

Is the statement about the slush in the entryway
admissible?

(A) No, because it is a statement made in the course of compromise negotiations.
(B) No, because the manager denied that the slippery condition was the cause of the plaintiff’s fall.
(C) Yes, as a statement by an agent about a matter within the scope of his authority.
(D) Yes, because the rule excluding offers of compromise does not protect statements of fact made during compromise negotiations

A

(A) No, because it is a statement made in the course of compromise negotiations.

Note: protects not only offers of compromise, but also conduct or statements made in the course of compromise negotiations. The rationale is to allow the parties and counsel to speak freely during settlement negotiations, without having to worry that their statements will be used against them at trial.

63

Q

At a civil trial for slander, the plaintiff showed that the defendant had called the plaintiff a thief. In defense, the defendant called a witness to testify, “I have been the plaintiff’s neighbor for many years, and people in our community generally have said that he is a thief.”
Is the testimony concerning the plaintiff’s reputation in the community admissible?

(A) No, because character is an essential element of the defense, and proof must be made by specific instances of conduct.
(B) Yes, to prove that the plaintiff is a thief, and to reduce or refute the damages claimed.
(C) Yes, to prove that the plaintiff is a thief, but not on the issue of damages.
(D) Yes, to reduce or refute the damages claimed, but not to prove that the plaintiff is a thief.

A

B) Yes, to prove that the plaintiff is a thief, and to reduce or refute the damages claimed.

In slander cases, where the defendant makes a statement that the plaintiff has an unsavory character, the plaintiff’s character is considered “in issue” (i.e., an essential element of the claim or defense under the substantive law) in two respects:

1) the plaintiff’s actual character will determine whether the defendant was incorrect in his assessment, and thus liable for slander, because truth is a defense.

2) the plaintiff will allege that he has been damaged by the statement, which is another way of saying that his true character has been besmirched; but if the plaintiff actually has a bad reputation anyway, then damages are limited.

64

Q

The director of a company that produces organic products was being sued by the minority shareholders of the company for allegedly breaching his duty of care and unwisely investing corporate funds into an organic clothing venture. During the trial, a witness for the minority shareholders testified that the director had been accused of battery three years ago, although he was never charged or arrested for the crime. The director’s lawyer then attempted to call a witness to testify that the witness heard the individual who had accused the director of battery verbally retract his accusation. The lawyer for the minority shareholders objected to the introduction of this evidence. The court overruled the objection and allowed the testimony.

Did the court err by overruling this objection?

A) No, because the admission of additional evidence refuting the battery accusation was necessary to remove any unfair prejudice.
B) No, because a court can always allow additional evidence to rebut previously admitted inadmissible evidence.
C) Yes, because the evidence of the retraction of the battery accusation was not relevant to the lawsuit.
D) Yes, because the individual’s retraction of the battery accusation constitutes hearsay.

A

A) No, because the admission of additional evidence refuting the battery accusation was necessary to remove any unfair prejudice.

Under the doctrine of curative admission, when inadmissible evidence is improperly admitted against a party, the court may permit that party to introduce additional inadmissible evidence for the purpose of rebuttal. This is meant to remedy the prejudicial effect caused by the previously admitted evidence.

Note: Here, the testimony that the director was once accused of battery was improper because it is not relevant to whether he breached his duty of care to the minority shareholders.

The director then offered testimony from a witness who heard the alleged battery victim recant his accusation. Although that evidence is hearsay—and also not relevant to the lawsuit—it was necessary to remove the unfair prejudice caused by the initial improper evidence

65

Q

A man is on trial for the misdemeanor crime of public lewd conduct for allegedly streaking through a gym in the early evening of May 5. This crime is not classified as a sexual offense in the jurisdiction. At trial, a trainer at the gym identified the man as the streaker and testified that, as the man ran through the gym, he cheered for a professional football team that had recently made the playoffs. The man, however, testified that he was nowhere near the gym on the day in question. In rebuttal, the prosecution seeks to call the manager of another local gym to testify that on the afternoon of May 4, the man had streaked through her gym while cheering for the same football team. The defense has objected to the manager’s testimony.

Should the manager’s testimony be admitted?

A) No, because a prior bad act is not admissible to show that the man possessed a character trait in accord with which he acted on May 5.
B) No, because character may generally be proven only by reputation or opinion evidence, not specific acts.
C) Yes, to demonstrate the man’s propensity for streaking through gyms.
D) Yes, to identify the man as the person who streaked through the gym on May 5.

A

D) Yes, to identify the man as the person who streaked through the gym on May 5.

MIMIC: Evidence of a criminal defendant’s prior crimes or bad acts is inadmissible to show the defendant’s propensity to commit the charged crime. But it may be admissible for other purposes, like his identity

66

Q

A defendant was charged with possession of prescription drugs without a prescription. At trial, the defendant testified that he received the drugs from a friend and believed they were an acceptable over-the-counter medication. The prosecutor intends to ask the defendant about his previous three misdemeanor convictions in the last five years for possession of the same type of prescription drugs without a prescription.

Should the court allow the prosecutor to ask about the defendant’s previous convictions?

A) No, because the convictions are inadmissible character evidence.
B) No, because the convictions were misdemeanors.
C) Yes, to impeach the defendant’s character for truthfulness.
D) Yes, to show that the defendant knew the prescription drugs were not an over-the-counter medication.

A

D) Yes, to show that the defendant knew the prescription drugs were not an over-the-counter medication.

Evidence that a criminal defendant previously committed a similar crime or bad act is inadmissible character evidence if it is offered to show the defendant’s propensity to commit the charged crime. But that evidence may be admissible for other relevant, noncharacter purposes—e.g., proving absence of mistake.

67

Q

A defendant was charged with illegal possession of a firearm by a felon. At trial, a prosecution witness testified that he and the defendant were involved in a verbal altercation during which the defendant pulled a pistol. The witness further testified that the defendant shot at him with the pistol and that he returned the defendant’s fire. On cross-examination, the defendant’s attorney asked the witness if it was true that he, and not the defendant, fired first. The witness maintained that the defendant shot first. While the witness was subject to recall, the defendant called a police officer to the stand in order to impeach the witness by testifying that the witness told her that he, not the defendant, had fired first. The prosecution has objected to the officer’s testimony.

Should the court exclude the officer’s testimony as to the witness’s prior statement?

A) No, because it relates to the witness’s prior inconsistent statement.
B) No, because the witness is subject to recall.
C) Yes, because it involves a collateral matter.
D) Yes, because it is inadmissible hearsay

A

C) Yes, because it involves a collateral matter.

A party generally may not impeach the credibility of a witness by introducing extrinsic evidence of a collateral matter. Instead, the party must accept the witness’s testimony.

Here, the defendant sought to impeach the witness by introducing his prior inconsistent statement through the officer’s testimony (extrinsic evidence). The witness was subject to recall, so he could explain or deny the statement and the prosecution could question him about it. However, the issue of who fired first is collateral to the crime with which the defendant was charged—illegal possession of a firearm by a felon

68

Q

A defendant is alleged to have been the getaway driver in a bank robbery. The prosecution has introduced evidence that a bystander made the following excited utterance as the bank robbers fled the scene: “They jumped into that black car and drove away like maniacs!” The defendant wants to call a valet from the hotel across from the bank to testify that when the bystander and the valet discussed the event the next day, the bystander told the valet that the bank robbers got into a gray car. The bystander died of natural causes shortly after speaking to the valet.

Should the bystander’s statement to the valet be admitted?

A) No, because it was not made under penalty of perjury.
B) No, because the bystander is unavailable to testify.
C) Yes, but only to impeach the bystander.
D) Yes, to impeach the bystander and as substantive evidence that the getaway car was gray.

A

C) Yes, but only to impeach the bystander.

A hearsay declarant’s credibility may be attacked using any method allowed to impeach a testifying witness, including offering evidence of the declarant’s inconsistent statement. But that statement may not be used as substantive evidence unless it is also excepted or excluded from the hearsay rule.

Note: A hearsay declarant’s inconsistent statement need not have been made under penalty of perjury to be admitted for impeachment purposes.

69

Q

A church is being sued for negligent hiring after its bus driver got in a drunk-driving accident while driving a bus full of parishioners to a church retreat. Evidence discovered by the plaintiff suggests that the bus driver had a well-documented drinking problem. The defense called a witness from the bus driver’s last employer, who testified that no one at the driver’s last job was aware that the bus driver had a drinking problem. The plaintiff’s attorney asked on cross-examination whether the witness is a member of the defendant-church. The defense objects to the question as an improper impeachment question.

Is the plaintiff’s attorney’s question proper?

A) No, because evidence of a witness’s religious belief is not admissible to impeach the witness’s credibility.
B) No, because it would violate the witness’s First Amendment rights.
C) Yes, as proper impeachment evidence.
D) Yes, because the witness’s religious belief is relevant to her credibility under oath.

A

C) Yes, as proper impeachment evidence.

Bias: One method of establishing bias is showing that the witness is affiliated with the opposing party outside the context of the litigation. Such affiliation can be established through intrinsic or extrinsic evidence.

70

Q

An attorney represented both a retailer and a manufacturer in a pending products liability action against them. An investigator employed by the attorney attended a meeting between the attorney, retailer, and manufacturer regarding the upcoming trial. At trial, the plaintiff called the investigator to testify regarding statements made by the manufacturer to the investigator during the meeting in which the manufacturer had admitted liability. The manufacturer objected to this testimony on the grounds that the testimony was hearsay and the statements were protected by the attorney-client privilege.

How should the court rule on this objection?

A) Overrule the objection, because the manufacturer is an opposing party.
B) Overrule the objection, because the manufacturer’s statements were made to the investigator.
C) Sustain the objection, because of the attorney-client privilege.
D) Sustain the objection, because the statements are hearsay.

A

C) Sustain the objection, because of the attorney-client privilege.

Statements made by and offered against an opposing party are nonhearsay. Additionally, the attorney-client privilege protects confidential communications made for the purpose of obtaining or providing legal assistance for the client and extends to the attorney’s agents.

Here, communications made before a coclient (the retailer) remain protected BUT the manufacturer’s statements to the investigator would not be protected by the attorney-client privilege in a subsequent action between the manufacturer and the retailer.

71

Q

A consumer filed a products liability action in federal court on the basis of diversity jurisdiction. In complying with a discovery request, the defendant’s lawyer inadvertently, despite the lawyer’s reasonable efforts to protect privileged material, included a report prepared at the lawyer’s request by an investigator who was employed in anticipation of the litigation and was not expected to be called as a witness at trial.

Immediately upon learning of the mistake, the defendant’s lawyer sought to retrieve the report from opposing counsel, but the plaintiff’s lawyer refused, asserting that the defendant’s lawyer’s inclusion of the report in the discovery material constituted a waiver of the protection for the material. The defendant’s lawyer contended that the federal inadvertent-waiver rule applied to this disclosure.

Is the defendant’s lawyer correct?

A) No, because the federal inadvertent-waiver rule applies only to disclosures made to a federal office or agency.
B) No, because the federal inadvertent-waiver rule does not apply to a federal case based on diversity jurisdiction.
C) Yes, because the federal inadvertent-waiver rule applies to the disclosure of communications protected by the attorney-client privilege.
D) Yes, because the federal inadvertent-waiver rule applies to the disclosure of information covered by the attorney work-product doctrine.

A

D) Yes, because the federal inadvertent-waiver rule applies to the disclosure of information covered by the attorney work-product doctrine.

Federal inadvertent-waiver rule, an inadvertent disclosure of protected information does not waive the attorney-client privilege or the attorney work-product doctrine if:
(1) the disclosure was in a federal proceeding or to a federal agency and
(2) the privilege holder attempted to prevent disclosure and promptly tried to rectify the error.

Here, the report was inadvertently disclosed in a federal case, and the defendant’s lawyer had taken reasonable steps to prevent disclosure and immediately sought to retrieve it. This satisfies the four requirements of the federal inadvertent-waiver rule. Therefore, the defendant’s lawyer correctly contended that this rule applied to his disclosure of the report.

Note: The federal inadvertent-waiver rule does apply to the disclosure of communications protected by the attorney-client privilege

72

Q

n a sexual harassment action brought by an employee against her employer, the employee alleged that her supervisor had created a hostile work environment by making repeated crude and explicit sexual comments that were unwelcome. The employer filed a motion to admit evidence that the employee had a sexual relationship with her previous supervisor to show that the employee welcomed the employer’s advances.

How should the court rule on the admissibility of this evidence?

A) The court should admit evidence of the relationship because the exclusion of evidence offered to prove a victim’s sexual conduct or predisposition applies only to a criminal case.
B) The court should admit evidence of the relationship only if its probative value substantially outweighs the danger of harm and unfair prejudice to the employee.
C) The court should refuse to admit evidence of the relationship because evidence offered to prove a victim’s sexual conduct or predisposition is inadmissible in a civil case.
D) The court should refuse to admit evidence of the relationship only if its probative value is substantially outweighed by the danger of harm and unfair prejudice to the employee.

A

B) The court should admit evidence of the relationship only if its probative value substantially outweighs the danger of harm and unfair prejudice to the employee.

Evidence of a victim’s other sexual behavior or sexual predisposition is generally inadmissible in a civil proceeding involving sexual misconduct. But such evidence may be admitted if the court determines that the probative value of the evidence substantially outweighs the danger of harm to the victim and unfair prejudice to any party.

Note: the default position is that this evidence is inadmissible, and the court need not determine that its probative value is substantially outweighed by the relevant dangers before excluding the evidence.

73

Q

A woman sues a jewelry company for injuries stemming from an allergic reaction she had to a metal in a bracelet that she believes was manufactured by the jewelry company. The jewelry company alleges that the bracelet is a forgery made by another manufacturer and that the company is not liable. The woman seeks to introduce the following pieces of evidence: (1) trademark registrations with accompanying photographs indicating that the bracelet that allegedly caused her injuries bears the jewelry company’s trademark, (2) a bracelet the jewelry company acknowledges as its own for the jury to compare with the woman’s bracelet, (3) purchase orders for the jewelry company’s bracelets from the store from which the woman purchased her bracelet, and (4) a judgment obtained by another plaintiff against the jewelry company for a similar reaction to a bracelet made by the jewelry company.

Which of these is LEAST likely to be admissible on the issue of whether the bracelet is a forgery?

A) The evidence concerning the jewelry company’s trademark on the bracelet.
B) The evidence that the store purchased bracelets from the jewelry company.
C) The jewelry company’s bracelet, for comparison to the woman’s bracelet.
D) The judgment against the jewelry company obtained by another plaintiff

A

D) The judgment against the jewelry company obtained by another plaintiff

There is no specific hearsay exception for civil judgments.

Note: The public records exception applies to records of a public office that set out a matter observed pursuant to a legal duty to report. Since trademark registrations are required by law to be recorded in a principal registry by the U.S. Patent and Trademark Office, the registrations are admissible.

74

Q

A plaintiff who had been injured in a car accident with a truck brought an action against the employer of the truck driver for negligent hiring. Prior to trial, the employer filed a motion for summary judgment arguing that it was entitled to judgment as a matter of law. In her response, the plaintiff submitted an affidavit by a former secretary of the employer stating that the secretary overheard the truck driver tell the employer that he had a history of accidents while interviewing for the job.

Can the court properly consider the driver’s statement in the affidavit in ruling on the employer’s summary-judgment motion?

A) No, because the affidavit constitutes double hearsay.
B) No, because the statement constitutes hearsay.
C) Yes, because it is an opposing party’s statement.
D) Yes, because it is not hearsay.

A

D) Yes, because it is not hearsay.

An out-of-court statement is not hearsay if offered for a purpose other than to prove the truth of the matter asserted.

Here, the secretary’s affidavit stated that the secretary heard the driver disclose a history of accidents to the employer. That statement is hearsay if offered for the truth of the matter asserted—because no exclusion or exception applies. But it is not hearsay if offered to show that the employer had notice of the driver’s history and may have been negligent in hiring him

75

Q

An artist is on trial for arson of an art dealer’s gallery containing a number of pieces of artwork by the artist. The artist had a contract with the art dealer making the art dealer liable to the artist for any damage to her artwork under his care. The prosecution wants to admit the artist’s notarized contract with the art dealer. The prosecution also wants to have a bartender testify that the artist told him, “I know I’m a terrible artist. Maybe the art dealer can sell a piece or two, but I’ll make more money by burning the place to the ground.” The artist has moved to exclude the contract for lack of notice and the statement to the bartender as inadmissible hearsay.

How should the court rule on the artist’s motions?

A) Exclude both pieces of evidence.
B) Exclude only the contract.
C) Exclude only the statement to the bartender.
D) Admit both pieces of evidence.

A

D) Admit both pieces of evidence.

Notarized documents are self-authenticating and can be introduced without advance notice to an adverse party. Additionally, statements made by and offered against a party-opponent are excluded from the hearsay rule.

76

Q

A plaintiff and a defendant were involved in a car accident in an intersection. Both parties sustained minor injuries in the accident, and the plaintiff subsequently brought an action for negligence against the defendant. Although there was evidence that both parties may have been negligent, the plaintiff sought to establish that the defendant’s vehicle did not slow down when it entered the intersection.

The plaintiff’s attorney called a passenger who was in the defendant’s vehicle at the time of the accident. The passenger testified that when she and the defendant got out of the car after the accident, the defendant told her, “It felt like the brakes failed, but that’s not possible, my mom just had the car serviced.” The defendant objected to the admission of this testimony.

Should the court sustain the defendant’s objection to the passenger’s testimony?

A) No, because the testimony can be offered by the plaintiff as a statement of a party-opponent.
B) No, because the testimony is only admissible for impeachment purposes.
C) Yes, because the defendant’s statement to the passenger was not based on personal knowledge.
D) Yes, because the defendant’s statement was not against his interest at the time it was made.

A

A) No, because the testimony can be offered by the plaintiff as a statement of a party-opponent.

Here, the statement was made by and offered against the defendant, so it constitutes a nonhearsay statement by a party-opponent. As a result, it is admissible substantively to prove that the defendant’s car did not slow down when it entered the intersection

77

Q

In a medical malpractice case, a patient sued her surgeon for allegedly causing organ damage during a routine surgery. In a previous case, the patient sued her medical insurance company for not covering some of the medical expenses associated with the complications from the organ damage. During that trial, the patient testified that she overheard a nurse assisting the surgeon tell another assisting nurse during the surgery that it appeared the patient’s kidney had been pierced. Before the patient could testify about this statement in the medical malpractice trial, she died from complications stemming from the surgery.

Is the patient’s testimony from the previous trial admissible in the medical malpractice trial?

A) No, because the defense did not have an opportunity to question the patient in the previous trial.
B) No, because the testimony was given during a previous trial, not the current proceeding.
C) Yes, because the patient is not available and the surgeon caused the patient’s unavailability.
D) Yes, because the patient is not available and the testimony was given as a witness at trial.

A

A) No, because the defense did not have an opportunity to question the patient in the previous trial

The former testimony hearsay exception allows admission of testimony (1) given at a trial, hearing, or deposition in the same case or a different proceeding and (2) offered against a party who—or whose predecessor in interest— had an opportunity and similar motive to develop that testimony.

Here, the patient died before she could testify in the medical malpractice trial and is therefore unavailable. But the surgeon did not have the opportunity to examine the patient during the previous trial. And the insurance company—whose objective at trial was to justify its denial of coverage and not to clear the surgeon of liability—is not the surgeon’s predecessor in interest.

78

Q

At the trial of a contract dispute, the plaintiff has offered to testify to what she heard the defendant say in a private conversation between the two of them, which the plaintiff secretly recorded on an audiotape that she did not offer in evidence.
Is the plaintiff’s testimony admissible?

(A) Yes, because the plaintiff has personal knowledge of the statement of a party-opponent.
(B) Yes, because the original document rule does not apply to audiotapes.
(C) No, because the statement must be proved by introduction of the audiotape itself.
(D) No, because of the plaintiff’s deception, even if the recording was not illegal.

A

(B) Yes, because the original document rule does not apply to audiotapes.

What the defendant said to the plaintiff, even in a private conversation, is a statement of a party-opponent and is admissible.
The plaintiff has personal knowledge of what the defendant said and can testify about it. The fact that the audiotape might be better evidence of what the defendant actually said makes no difference. The best evidence rule applies only when a witness testifies about the content of a writing or recording. Here the plaintiff would not be testifying about the content of the audiotape but rather about what she personally heard.

79

Q

A plaintiff sued a defendant for wrongful death arising out of a traffic collision between the plaintiff’s decedent and the defendant. At trial, the investigating police officer authenticated a tape recording of her shift-end dictation of comments used in preparing the written report of her factual findings. She has testified that the tape recording was accurate when made and that she currently has no clear memory of the details of the investigation.

Is the tape recording admissible as evidence?

(A) Yes, under the recorded recollection exception to the hearsay rule.
(B) Yes, under the public records exception to the hearsay rule.
(C) No, because it is hearsay and is a police report being offered against the defendant in a wrongful death case.
(D) No, because the police report itself is the best evidence.

A

(A) Yes, under the recorded recollection exception to the hearsay rule.

The witness once had knowledge but now has insufficient recollection to testify fully and accurately about her investigation. She made the recording when the matter was fresh in her memory, and she has testified that the recording was an accurate reflection of her memory.

Note: Although the officer’s formal written report would qualify as a public record, the informal, dictated comments she made to help her prepare the report would not. The tape recording is admissible, however, under the recorded recollection exception to the hearsay rule.

80

Q

A plaintiff sued an individual defendant for injuries suffered in a collision between the plaintiff’s car and the defendant’s truck while the defendant’s employee was driving the truck. The plaintiff sought discovery of any accident report the employee might have made to the defendant, but the defendant responded that no such report existed. Before trial, the defendant moved to preclude the plaintiff from asking the defendant in the presence of the jury whether he had destroyed such a report, because the defendant would then invoke his privilege against self-incrimination.

Should the court allow the plaintiff to ask the defendant about the destruction of the report?

(A) No, because a report that was prepared in anticipation of litigation is not subject to discovery.
(B) No, because no inference may properly be drawn from invocation of a legitimate privilege.
(C) Yes, because a party in a civil action may not invoke the privilege against self-incrimination.
(D) Yes, because the defendant’s destruction of the report would serve as the basis of an inference adverse to the defendant.

A

(D) Yes, because the defendant’s destruction of the report would serve as the basis of an inference adverse to the defendant.

If a party destroys evidence, it is proper for the jury to draw an inference that the evidence was adverse to that party’s case. It is also proper for the jury to draw an adverse inference in a civil case from a party’s assertion of the privilege against self-incrimination. Thus, the court should allow the question to be asked, because it is proper regardless of how the defendant responds.

Note: The privilege against self-incrimination may be asserted in both civil and criminal cases so long as the statement made in response to the question posed could tend to incriminate the person in a criminal prosecution.

81

Q

A plaintiff sued a defendant for injuries allegedly suffered when he slipped and fell on the defendant’s business property. At trial, without asking that the defendant’s property manager be declared a hostile witness, the plaintiff called him solely to establish that the defendant was the owner of the property where the plaintiff fell. On cross-examination of the manager, the defendant’s attorney sought to establish that the defendant had taken reasonable precautions to make the property safe for business invitees.
Should the defendant’s cross-examination of the manager be permitted over the plaintiff’s objection?

(A) No, because cross-examination should be limited to the subject matter of the direct examination and matters affecting the credibility of the witness.
(B) No, because the court has not declared the manager hostile.
(C) Yes, because the cross-examiner is entitled to explore matters relevant to any issue in the case, including credibility.
(D) Yes, because the manager is the agent of a party, as to whom the scope of cross-examination is unlimited.

A

(A) No, because cross-examination should be limited to the subject matter of the direct examination and matters affecting the credibility of the witness.

Note: cross-examination should be limited to the subject matter of the direct examination and matters affecting credibility. Although the court has discretion under this rule to permit inquiry into additional matters, the defendant is not “entitled” to a wider scope of cross- examination.

82

Q

A defendant was charged with assault after being involved in a barroom fight in the middle of the day. The defendant admitted to being at the bar at the time of the fight, but claimed that he was only a bystander. At the defendant’s trial, the prosecutor intended to call the defendant’s former employer. The employer was to testify that the defendant had been fired and was not working at the time of his arrest. The defendant objected to the employer’s testimony.

How should the court rule?

A) Overrule the objection, because it tends to make it more likely that the defendant was at the bar in the middle of the day and involved in the fight.
B) Overrule the objection, because the employer will be testifying based on his personal knowledge of the defendant’s employment.
C) Sustain the objection on the basis that the employer’s testimony is unfairly prejudicial.
D) Sustain the objection on the basis that the employer’s testimony is not probative of a material fact.

A

C) Sustain the objection on the basis that the employer’s testimony is unfairly prejudicial.

Relevant evidence may be excluded if its probative value is substantially outweighed by the danger of unfair prejudice. Here, the employer’s testimony includes the fact that the employee was fired. This fact is highly prejudicial, as it would tend to discredit the defendant in the minds of the jury. Moreover, the probative value of the employer’s testimony is somewhat minimal.

Note: the employer’s testimony regarding the defendant’s unemployment is relevant to whether the defendant was present at the bar (and therefore potentially involved in the fight). The mere fact that the defendant has stipulated to his presence at the bar does not automatically preclude the prosecution from presenting the employer’s testimony, which also suggests the defendant was at the bar.

83

Q

While on her morning run, a runner was struck by a driver’s vehicle as she crossed the street, suffering a broken arm in the resulting fall to the ground. The driver was allegedly exceeding the speed limit because he was late for work. In a civil action for negligence against the driver, the runner alleges that the driver’s speeding at the time of the accident constitutes negligence per se, but the driver denies that he was speeding. The runner intends to introduce the testimony of the driver’s neighbor that the driver has a reputation for speeding regularly in their neighborhood.

Should the court allow the runner to introduce the proposed testimony of the driver’s neighbor?

A) No, because it constitutes improper character evidence.
B) No, because the driver’s tendency for speeding can only be established through specific instances of conduct.
C) Yes, because the testimony is admissible to show that the driver was likely speeding at the time of the accident.
D) Yes, because the driver’s reputation for speeding is an essential element of the runner’s claim against the driver.

A

A) No, because it constitutes improper character evidence.

In a civil case, evidence of a person’s character (or character trait) generally is inadmissible to prove that the person acted in accordance with that character (or character trait) on a particular occasion. In this case, the runner is attempting to admit testimony regarding the driver’s reputation for speeding in order to show that the driver was likely speeding at the time of the accident

84

Q

defendant was on trial for murder. The pastor of his church was called as a character witness. The pastor testified that the defendant has a reputation in the community for peacefulness. On cross-examination, the prosecutor, seeking to impeach the pastor and having a good-faith basis for his question, asked the pastor if he had a violent temper.

Of the following, which is the defense attorney’s best basis for objecting to this impeachment question?

A) The question exceeds the scope of the direct examination of the pastor.
B) The question does not relate to the defendant’s character.
C) The question calls for impermissible character evidence.
D) The question violates the limitation imposed on evidence of a witness’s religious beliefs or opinions.

A

C) The question calls for impermissible character evidence.

The prosecutor is attempting to show that the pastor, a violent person, is a bad person and therefore should not be believed. Because this is not directly related to the pastor’s truthfulness, it is impermissible character evidence.

Note: any witness may be impeached by certain types of character evidence

85

Q

At his trial for larceny, a defendant called his brother as a character witness. The brother testified that the defendant had a reputation in the community for being an honest man. During rebuttal, the prosecutor called the defendant’s former employer to testify that the defendant lied on his job application. The defense attorney objected, arguing that the testimony is an improper use of character evidence.

How should the judge rule on the defense attorney’s objection?

A) Sustain the objection, because the evidence has no probative value on any issue in the case.
B) Sustain the objection, because the evidence can be inquired into only during cross-examination of the brother.
C) Overrule the objection, because the defense has “opened the door” to the prosecutor’s admission of this evidence.
D) Overrule the objection, because the evidence offered involves dishonesty.

A

B) Sustain the objection, because the evidence can be inquired into only during cross-examination of the brother.

on cross-examination, the prosecution may question a defendant’s character witness about specific instances of the defendant’s conduct. Here, the prosecution could only introduce the specific instance of conduct by **asking ** the brother about the conduct on cross-examination. This evidence cannot be introduced by extrinsic evidence

Note: Although the defendant “opened the door” by having his brother testify as to his character, thus permitting the prosecution to rebut that testimony, the prosecution is limited to asking the brother about the specific instance of conduct on cross-examination, and cannot introduce extrinsic evidence of the defendant lying on a job application

86

Q

A baker and a chef ran a catering business as a limited liability company (LLC) with the two as the only members of the LLC, and the chef as the managing member. Recently, the chef sought an equitable accounting of the assets of the LLC. The court referred the matter to a referee to hear evidence and make findings of facts with regard to the matter. One matter of contention was how much money the baker had received from the business’s proceeds in cash. The baker testified that each time he took money from the business, he logged the withdrawals in a notebook, which was offered into evidence by the baker without objection from the chef. On the witness stand, the chef admitted that she had access to the notebook, which was kept in a safe on the premises of the business, and never made nor altered any entries in it. She further testified that on several specific dates, she remembers watching the baker take $500 cash from the business. This was a larger sum of money than the amount recorded in the notebook admitted into evidence. The baker immediately objected.

Is the chef’s testimony admissible?

A) No, because the best evidence rule will not allow the baker to contradict the notebook.
B) No, because the contents of the notebook are at issue.
C) Yes, because the notebook is not a document with legal effect.
D) Yes, because the testimony is based on the chef’s firsthand knowledge.

A

D) Yes, because the testimony is based on the chef’s firsthand knowledge.

A non-expert witness must have personal knowledge of a matter in order to testify about that matter. Personal knowledge may be established by the witness’s own testimony as well as through other means.

Note: Whether the contents are at issue does not impact the chef’s ability to introduce evidence based on his personal knowledge that contradicts the document.

87

Q

A defendant in a federal securities case introduced the testimony of a witness who had claimed on direct examination that the defendant had no prior knowledge of a change within a corporation’s executive board; the defendant’s knowledge of this fact was a central issue in the case. The prosecutor did not cross-examine the witness. On rebuttal, to impeach the defendant’s witness, the prosecutor called a witness who testified that she had heard the defendant’s witness say that the defendant knew of the change. Further, the prosecutor introduced a properly authenticated email that the defendant’s witness had sent to the witness containing the same information. The defendant’s attorney objects on the grounds that the testimony of the prosecutor’s witness and the email are inadmissible to impeach the defendant’s witness.

Should the court admit the testimony of the prosecutor’s witness and the email?

A) Yes, because the defendant’s witness may be properly impeached with them.
B) Yes as to the testimony, but no as to the email, because the prosecutor did not present the email to the defendant’s witness on her cross-examination of him.
C) No, because the defendant’s witness was not given an opportunity to explain the evidence before introduction of the prosecutor’s witness.
D) No, because the testimony and email are immaterial.

A

A) Yes, because the defendant’s witness may be properly impeached with them.

Because these statements are being used to impeach the witness and not to prove the truth of the matter asserted, they are not hearsay. Also, because the statements were not made under oath in a prior proceeding, they may not be considered as substantive evidence

88

Q

A customer filed suit against a coffee maker’s manufacturer for injuries she sustained when the coffee maker exploded and severely injured her, under both negligence and strict liability theories. The judge granted summary judgment in favor of the manufacturer on the negligence claim and granted summary judgment in favor of the customer on the strict liability claim. The case then proceeded to jury trial on the issue of damages only. At trial, the customer qualified an expert witness who opined that there was a reasonable alternative design for the coffee maker and that the manufacturer’s failure to use that design probably rendered the product not reasonably safe. The manufacturer objects to the expert’s testimony.

What is the strongest basis for the manufacturer’s objection?

A) The testimony needlessly presents cumulative evidence.
B) The testimony concerns an ultimate issue in this case.
C) The testimony is irrelevant.
D) The expert does not possess the requisite degree of certainty in his opinion.

A

C) The testimony is irrelevant.

The judge has already ruled on the negligence and strict liability issues, and the only remaining issue is the amount of damages to which the customer is entitled. Consequently, the expert’s testimony as to whether there was design defect in the coffee maker is not relevant to this issue

Note: The issue of whether there was a reasonable alternative design was never before the jury because of the judge’s ruling at the summary judgment stage. Because the jury has not heard any other evidence on this issue, the expert’s testimony on the subject is not cumulative.

89

Q

A plaintiff sued a defendant for fraudulent representations allegedly made by the defendant in selling an automobile to the plaintiff. At trial, the plaintiff testified that he received a letter from the defendant the day before the sale occurred in which the defendant wrote that the brakes on the automobile had just been replaced the week before. The defendant objected to the testimony.

May the court properly admit the testimony over the defendant’s objection?

A) No, because the statement constitutes hearsay.
B) No, because the letter was not properly authenticated.
C) No, because the best evidence rule applies.
D) Yes, because the statement does not constitute hearsay.

A

C) No, because the best evidence rule applies.

Here, the plaintiff is attempting to prove the contents of the letter sent by the defendant. There is no indication that the original is lost, destroyed, or in the possession of the defendant. The matter is not collateral, but central to the question of whether fraud occurred.

Note: the plaintiff was not actually trying to put the letter into evidence. Only when a document is being placed into evidence would it have to be authenticated.

90

Q

A defendant was convicted of cocaine distribution. At the defendant’s sentencing hearing, the prosecution called the defendant’s wife to testify regarding a statement made by the defendant about the amount of cocaine the defendant had purchased from a supplier. The amount of cocaine possessed by the defendant was relevant to the length of defendant’s sentence. The defendant objected on the grounds that the statement was made during a private conversation on their honeymoon.

Should the court permit the defendant’s wife to testify as to his prior statement?

A) No, because the statement is hearsay.
B) No, because the statement is privileged.
C) Yes, because the Federal Rules of Evidence do not apply to sentencing proceedings.
D) Yes, because the testifying spouse holds the privilege not to testify regarding the communication.

A

B) No, because the statement is privileged.

Note: While the Federal Rules of Evidence generally do not apply to sentencing proceedings, the rules regarding privileges apply to all federal court cases and proceedings, including sentencing proceedings.

91

Q

A lawyer represented both an internist and a hospital that employed him in a pending action against them based on the alleged negligence of the internist in treating a patient. A stenographer employed by the attorney attended a meeting between the lawyer, the internist, and the hospital. The stenographer recorded what was said at the meeting. Subsequently, the hospital filed a cross-claim against the internist seeking indemnity for any damages assessed against it. The lawyer was compelled to withdraw her representation of both parties before trial. At trial, the internist called the stenographer to testify regarding a statement made by the hospital’s president during the meeting that the hospital would “back the internist 100 percent.” The hospital, asserting the attorney-client privilege, objected to this testimony.

Should the court overrule this objection?

A) Yes, because the lawyer no longer represents either party.
B) Yes, because the dispute involves the hospital and the internist.
C) No, because the stenographer was an employee of the lawyer.
D) No, because the presence of the stenographer did not eliminate the confidentiality of the statement.

A

B) Yes, because the dispute involves the hospital and the internist.

Confidential communications made by a client for the purpose of obtaining or providing legal assistance for the client is generally protected by the attorney-client privilege. BUT this privilege does not apply to communications between former co-clients who are now adverse to each other.

92

Q

During a defendant’s criminal trial for assault, a witness for the defendant testified that the defendant had told her that he had never even met the victim. The defendant was acquitted. Subsequently, the victim brought a civil assault claim against the same defendant based on the same alleged assault. The same witness was called as a witness for the victim, and she testified that, shortly after the assault took place, the defendant had admitted to her that he did assault the victim. On cross-examination, the defendant’s counsel asked the witness whether she had testified in the defendant’s prior criminal prosecution that the defendant told her that he had never even met the victim. The victim’s attorney objected to the defendant’s question.

May the court allow the question over the objection?

A) Yes, but only to prove that the defendant did not commit the assault.
B) Yes, but only to impeach the credibility of the witness.
C) Yes, both to prove that the defendant did not commit the assault and to impeach the credibility of the witness.
D) No, because the defendant did not first allow the witness the opportunity to explain or deny the statement.

A

C) Yes, both to prove that the defendant did not commit the assault and to impeach the credibility of the witness.

The witness’s prior inconsistent statement is admissible to impeach the witness’s credibility. The witness’s prior testimony is also admissible substantively as non-hearsay because her statement was made under oath at the prior criminal proceeding.

93

Q

At a criminal trial for rape, the prosecution seeks to admit into evidence a statement made by the defendant to his friend, acknowledging that he raped the victim. The defendant made the statement shortly after the rape, but before he knew he was a suspect. The defendant testified at trial that he did not rape the victim.

Is the statement the defendant made to his friend acknowledging the rape admissible?

A) Yes, the statement is admissible as non-hearsay.
B) Yes, the statement is admissible because it falls within the statement against interest exception to the hearsay rule.
C) No, the statement is inadmissible hearsay not within any exception.
D) No, because the statement was not against the defendant’s interest at the time he made it.

A

A) Yes, the statement is admissible as non-hearsay.

Opposing Party Statement: a statement made by a party to the current litigation is not hearsay if it is offered by an opposing party.

Here, the defendant is a party to the current litigation, and the prosecution is seeking to use the defendant’s own statement against him.

Note: statement against interest exception to the hearsay rule only applies if the declarant is unavailable to testify at the present trial

94

Q

A defendant was charged with involuntary manslaughter. The defendant testified that he did not shove the victim, but that instead she had tripped and fell down the stairs. Subsequently, the prosecution called a police officer who had talked with the defendant at the scene of the incident. The police officer testified that, on the day of the incident, the defendant told the officer that he had shoved the victim before she fell down the stairs. The prosecution had not asked the defendant about this statement when he was on the witness stand.

Is this statement admissible as substantive evidence of the defendant’s actions?

A) No, because the prosecution did not ask the defendant about the statement when the defendant was on the witness stand.
B) No, because the statement was not made under oath.
C) Yes, because it contradicts the defendant’s testimony.
D) Yes, because the defendant made the statement to the police officer.

A

D) Yes, because the defendant made the statement to the police officer.

A statement made by a party to the current litigation is not hearsay if it is offered by an opposing party, even though the statement is being offered to prove the truth of the matter asserted. Consequently, the defendant’s statement to the police officer that the defendant shoved the victim before she fell down the stairs is admissible, even though if made by someone else, it would be inadmissible hearsay

Note: Although a prior inconsistent statement made by a witness generally must have been made under oath in order to be admitted as substantive evidence, this is not the case with regard to an opposing party’s statement

95

Q

The owner of a dog was inside while her dog was outside in a fenced-in yard. Hearing a child crying, the owner looked out a window and noticed her neighbor’s three-year-old child inside the owner’s yard. The owner went outside and discovered that the child’s face had been deeply scratched by the owner’s dog. The owner took the child to the neighbor’s home, telling the child’s mother, “My dog scratched your child’s face. I’m liable for his injuries.”

Are either of the owner’s statements admissible at trial in an action brought by the child’s parents on the child’s behalf against the owner to recover damages?

A) No, as to both statements.
B) Yes, but only the first statement because the second statement is an opinion by a lay witness about the ultimate issue in the case.
C) Yes, but only the second statement because the owner did not personally witness the event.
D) Yes, both statements are admissible because they were made by the owner.

A

D) Yes, both statements are admissible because they were made by the owner.

A statement made by a party to the current litigation is not hearsay if it is offered by an opposing party.

Note: An opposing party’s statement in the form of an opinion may be admitted even if the statement is about a matter that normally would be beyond the scope of lay witness opinion testimony, such as an opinion regarding the ultimate issue in the case.

96

Q

A man was on trial for bank robbery and felony murder. The prosecution alleged that while the man went into a bank wearing a ski mask and wielding a gun, his getaway driver waited in a car outside. The man shot a bank patron during the robbery then got into the getaway car with the money. In the high-speed chase that followed, the getaway car crashed and the getaway driver died.

At trial, the defense called a bartender who talked to the getaway driver the night before the robbery. The bartender testified that the getaway driver had told him that because the man refused to join him in the robbery unless they used a fake gun, the getaway driver had given him a real gun to use and told him it was fake. The court ruled that the statement was admissible as a statement against the getaway driver’s interest.

The prosecution now wishes to call a woman to testify that the getaway driver had told her a week before the robbery that the man had purchased a gun and was planning on robbing a bank.

Is the woman’s testimony admissible?

A) No, because the getaway driver had no opportunity to explain or deny the statement.
B) No, because the getaway driver’s statement was not made under oath.
C) Yes, to impeach the getaway driver.
D) Yes, under a hearsay exception for an unavailable declarant.

A

C) Yes, to impeach the getaway driver.

Here, the getaway driver’s hearsay statement—that he gave the man a real gun but told him it was fake because the man had demanded that they use a fake gun—was admitted as a statement against interest. Therefore, the prosecution can offer testimony regarding the getaway driver’s inconsistent statement to the woman—that the man had purchased a real gun to use during the robbery—for the purpose of impeaching the getaway driver.

97

Q

A defendant is on trial for felony assault. Eleven years ago, the defendant was convicted of rape. He was sentenced to three years imprisonment and served the full term. The defendant decided to testify at his current trial for felony assault, and on cross-examination, the prosecution seeks to admit evidence of the defendant’s rape conviction to impeach him.

Which of the following best states the standard that must be met to determine the admissibility of this conviction?

A) The conviction is admissible only if its probative value outweighs the prejudicial effect to the defendant.
B) The conviction is admissible only if its probative value substantially outweighs its prejudicial effect.
C) The court has discretion to exclude the conviction if the defense shows that its probative value is substantially outweighed by its prejudicial effect.
D) The court must exclude the conviction if the defense shows that its probative value is substantially outweighed by its prejudicial effect.

A

A) The conviction is admissible only if its probative value outweighs the prejudicial effect to the defendant.

A conviction for a felony not involving dishonesty that is not more than 10 years old is admissible against a criminal defendant-witness if the conviction’s probative value outweighs its prejudicial effect.

98

Q

The user of a power tool sued the tool’s manufacturer in state court. The action was based on a strict product liability claim that the manufacturer’s failure to adequately warn the user of a defect in the power tool caused the user’s injury. The manufacturer properly removed the case to federal court. The applicable law of the state that governs the existence of the strict product liability claim also recognizes a rebuttable heeding presumption. This presumption assumes that an injured plaintiff would have heeded an adequate warning if one had been given. Under state law, this presumption does not shift the burden of persuasion on this issue to the manufacturer.

The manufacturer did not present evidence that the user would not have heeded a different warning had it been given. The court instructed the jury that it must apply the presumption that the warning, if given, would have been heeded.

Is the court’s instruction correct?

A) No, because the jury may, but is not required to, apply the presumption.
B) No, because state law presumptions are not recognized in a federal diversity action.
C) Yes, because the manufacturer failed to offer evidence to rebut the presumption.
D) Yes, because the Federal Rules of Evidence apply the bursting-bubble approach to presumptions.

A

C) Yes, because the manufacturer failed to offer evidence to rebut the presumption.

A rebuttable presumption: conclusion that can be overcome if the opposing party produces sufficient contrary evidence. However, if no contrary evidence is produced, the judge should instruct the jury that it must apply the presumption

In federal cases in which state law supplies the rule of decision for a claim or defense (i.e., in diversity cases), it also governs the effect of presumptions related to the claim or defense.

Because the manufacturer failed to offer evidence to rebut the presumption that the user would have heeded a warning on the power tool, the court correctly instructed the jury that it must apply the presumption.

99

Q

A security guard was charged with criminal battery of a student. The prosecution alleged that the security guard used excessive force when he removed the student from a campus event after the student became disruptive. The security guard’s only argument in his defense was that he exerted lawful force to remove the disruptive student from the event pursuant to his duties as a security guard. The security guard testified that the student had a reputation on campus for disrupting campus events by starting physical fights with other students.

Is the security guard’s testimony regarding the student’s reputation admissible?

A) No, because the prosecution has not presented evidence of the security guard’s bad character.
B) No, because the security guard has not asserted a theory of self-defense.
C) Yes, because the security guard has personal knowledge of the student’s reputation for violence.
D) Yes, because the student’s reputation for violence is relevant to whether the student was the initial aggressor.

A

B) No, because the security guard has not asserted a theory of self-defense.

A criminal defendant may introduce evidence of an alleged victim’s character—but only if it is pertinent to the case.

Here, the security guard’s only defense to the criminal battery charge is that he exerted lawful force to remove the disruptive student from the campus event. In support of that defense, the guard sought to testify that the student has a reputation for disrupting campus events by starting physical fights with other students. Although that character evidence would be pertinent to a self-defense claim, it is not pertinent to whether the guard’s level of force was lawful

100

Q

A property owner constructed a fence along what he thought was the boundary to his property. A year later, a neighbor who was selling adjoining property had a survey conducted. As a consequence of the survey, the neighbor brought an action against the property owner seeking removal of the fence.

At trial, the neighbor testified that he had orally objected to the property owner about the placement of the fence at the time it was constructed. After the neighbor left the witness stand, the property owner sought to introduce into evidence a certified copy of the official judgment and conviction of the neighbor for perjury. The neighbor was convicted 11 years ago and released from prison nine years ago. The property owner had not asked the neighbor about the conviction while the neighbor was on the stand.

Should the court permit the introduction of the judgment for the purpose of impeaching the neighbor’s testimony?

A) No, because the neighbor was convicted of perjury more than 10 years ago.
B) No, because the neighbor was not questioned about the conviction while the neighbor was on the witness stand.
C) Yes, because a conviction used to impeach a witness’s character for truthfulness may be proved by extrinsic evidence.
D) Yes, because a witness in a civil case may not be impeached with a previous conviction.

A

C) Yes, because a conviction used to impeach a witness’s character for truthfulness may be proved by extrinsic evidence.

Convictions for crimes of dishonesty are automatically admissible to impeach any witness if the conviction—or the release from incarceration—is not more than 10 years old, and use of extrinsic evidence is allowed for that purpose. Therefore, the court should permit the introduction of the judgment.

101

Q

A plaintiff brought suit against a defendant for injuries she sustained in a car accident that she accused the defendant of negligently causing. Prior to filing suit, the plaintiff’s attorney had the plaintiff visit a physician to determine the extent of her injuries for purposes of determining the damages to be claimed in the lawsuit. After the plaintiff’s examination, while the attorney, plaintiff, and physician were discussing the extent of the plaintiff’s injuries, the plaintiff admitted that she “may have had a few beers” right before the accident. At trial, the defendant’s counsel sought to call the doctor to testify about the statement. The plaintiff properly objected to the introduction of this testimony.

How should the judge rule on the plaintiff’s objection?

A) Sustain the objection, as the attorney-client privilege is applicable.
B) Sustain the objection, as the physician-patient privilege is applicable.
C) Overrule the objection, as the statement was made by an opposing party.
D) Overrule the objection, as the physician would constitute an expert witness.

A

A) Sustain the objection, as the attorney-client privilege is applicable.

Here, the plaintiff’s attorney had the plaintiff visit a physician (an expert) to ascertain the extent of her injuries for purposes of determining the damages to be claimed in the lawsuit.

This means that the physician was hired to assist the attorney in the rendition of professional legal services by providing medical consultation. And since there is no indication that the physician was hired to testify at trial, the plaintiff’s statement in the physician’s presence remains protected by the attorney-client privilege

102

Q

A plaintiff was shot by the defendant while the two were hunting. The plaintiff filed a negligence action against the defendant to recover damages for injuries suffered as a result of the incident. In his case-in-chief, the plaintiff seeks to introduce the testimony of a mutual friend of both the plaintiff and the defendant that, in the friend’s opinion, the defendant is habitually a careless person. Is this evidence admissible?

A) Yes, because it is has a tendency to make it more probable that the defendant was negligent in shooting the plaintiff.
B) Yes, as habit evidence.
C) No, because it is inadmissible character evidence.
D) No, because character evidence may not be presented in the form of opinion testimony.

A

C) No, because it is inadmissible character evidence.

Here, evidence that the defendant was a careless person is not admissible to prove that the defendant was careless at the time the plaintiff was shot.

Note: Although the fact that the defendant is careless may be probative in that it has a tendency to make it more probable that the defendant was negligent in the matter at hand, character evidence is inadmissible to prove that a person acted in accordance with that character on a particular occasion – improper purpose.

103

Q

A customer at a grocery store slipped and fell on a wet spot on the floor. The customer sued the store to recover for the resulting injuries to her back. At trial, the store’s lawyer sought to prove that the damage to the customer’s back was not serious, and called the customer’s neighbor to testify regarding the severity of the customer’s injuries. The neighbor testified that on the morning after the fall, the customer walked to her car wearing a tennis dress and carrying a tennis racket. The neighbor then added, “I told my sister, ‘There goes [the customer] off for her weekly tennis match.’” The customer’s lawyer moves to the strike this statement as inadmissible hearsay.

How should the court rule on this motion?

A) Deny the motion, because, since the declarant is testifying, the statement is not hearsay.
B) Deny the motion, because the neighbor was describing an event as it was taking place.
C) Grant the motion, because the statement is being offered for its truth.
D) Grant the motion, because the sister can testify to the neighbor’s statement.

A

B) Deny the motion, because the neighbor was describing an event as it was taking place.

Present Sense Impression Exception: DESCRIBING event while perceiving it

104

Q

The defense attorney in a criminal rape case wants to introduce testimony that the alleged victim has a reputation in the community for promiscuity. The testimony will come from one of the victim’s past sexual partners. The prosecutor objects to the introduction of the testimony, but the defense maintains that because consent is at issue in the case, the testimony is relevant.

Should the court allow the testimony?

A) Yes, because reputation testimony is an admissible form of character evidence.
B) Yes, because the victim’s past sexual behavior can be used to prove consent.
C) No, because evidence of a victim’s past sexual behavior is inadmissible.
D) No, because this evidence of the victim’s other sexual behavior is not relevant.

A

D) No, because this evidence of the victim’s other sexual behavior is not relevant.

Some instances in which a victim’s past sexual behavior may be relevant and admissible:

1) evidence of a victim’s past sexual conduct is admissible to show the victim’s past sexual behavior with the defendant in order to prove consent

2) to prove an alternate source of sem*n or injury

3) when the constitutional rights of the defendant require admission of the evidence

4) Civil cases –> when the probative value substantially outweighs danger of harm to the victim

5) Civil Cases –> when the victim herself has placed her reputation in controversy.

105

Q

A few weeks before the beginning of a murder trial, a witness to the murder identified the defendant in a photo array as the person who killed the victim. Between the time of the photo array and the trial, the witness died. The witness was the only eyewitness to the crime, aside from the victim and the murderer, so the prosecution wants to admit at the trial the witness’s identification of the defendant from the photo array. The defendant’s attorney objects to the introduction of the statement, but the prosecutor claims that as a prior statement of identification, it is admissible.

Is the statement admissible as a prior statement of identification?

A) Yes, because prior statements of identification are admissible as non-hearsay.
B) Yes, because the witness is unavailable to testify.
C) No, because the witness did not testify at the present trial.
D) No, because the witness did not testify under oath at a previous trial, hearing, or deposition as to the identity of the murderer.

A

C) No, because the witness did not testify at the present trial.

A previous out-of-court identification of a person after perceiving that person is admissible as substantive evidence, but only if the witness testifies at the present trial or hearing and is subject to cross-examination concerning the identification

106

Q

Prior to trial, a criminal defendant filed a motion to prevent the prosecution from introducing statements by an alleged co-conspirator. At the hearing on the motion, the court definitively ruled that the statements could be admitted into evidence. At the jury trial, the prosecution called the alleged co-conspirator to testify about the statements. The defendant did not object to the introduction of the statements into evidence. On appeal, the defendant claimed that the trial court erred in admitting the statements.

Should the appellate court decide the merits of this claim of error?

A) No, because the defendant failed to object at trial when the prosecution introduced the statements.
B) No, because the admissibility of evidence is properly a question for a trial court.
C) Yes, because the court made a definitive ruling on the record admitting the statements.
D) Yes, because an objection was unnecessary under the plain error rule.

A

C) Yes, because the court made a definitive ruling on the record admitting the statements.

Once the court makes a definitive ruling on the admissibility of evidence, a party need not renew an objection to the admission of the evidence, even if the ruling was made before the trial began.

Note: Plain error rule allows a party to appeal after failing to object to the introduction of evidence.

It does not apply in this case because the defendant did object to the introduction of the statements before trial. Consequently, the appellate court’s review of the admissibility of the statements is not confined to the plain error rule.

107

Q

In a negligence action tried before a jury, a neighbor of the defendant who witnessed the plaintiff slip on the defendant’s sidewalk is called to testify as to what he saw. The neighbor, who normally wears glasses, did not have them on at the time of the incident. The neighbor is first questioned about the weather conditions at the time that the plaintiff slipped and about the plaintiff’s behavior before doing so. The neighbor is then asked whether the plaintiff’s conduct was negligent.

Of the following, which is the best ground upon which the opposing party can object to this question regarding negligence?

A) As a lay witness, the neighbor cannot testify as to an opinion.
B) The neighbor was not wearing his glasses at the time of the incident.
C) The neighbor’s opinion is not helpful to the jury’s clear understanding of his testimony.
D) The neighbor’s opinion is not rationally based on his perception.

A

C) The neighbor’s opinion is not helpful to the jury’s clear understanding of his testimony.

Here, the neighbor is being asked to give a legal conclusion without an understanding of the legal criteria for that conclusion. As such, the neighbor’s opinion is not helpful to the jury.

108

Q

A high school teacher played on a hockey team in a local recreational league. During a league game, the teacher was involved in a fight with another hockey player. That player sued the teacher in a battery action to recover for injuries inflicted during the fight. The teacher contended that he had acted in self-defense. The teacher called his principal to testify that the teacher had a reputation within the school community for peacefulness. The plaintiff, who had not introduced evidence of the teacher’s character for violence, objected to this testimony. Should the court admit this testimony?

A)Yes, because the defendant is entitled to introduce evidence of a pertinent good character trait.
B) Yes, because character evidence may be introduced through reputation testimony.
C) No, because the plaintiff had not introduced evidence of the teacher’s character for violence.
D) No, because such evidence is not admissible in a civil action.

A

D) No, because such evidence is not admissible in a civil action.

Student SUED the teacher = civil, not criminal case

Evidence of a defendant’s character is inadmissible in a civil case to prove that the defendant acted in conformity with that character trait unless the defendant’s character is an essential element of a claim or defense. Since the defendant’s character for peacefulness is not an element of either battery or self-defense, the principal’s testimony is not admissible.

Pertinent civil traits: negligent hiring, defamation, fraud, child custody

Note: a defendant is permitted to introduce evidence of a pertinent good character trait in a criminal case, such evidence is not admissible in a civil case.

109

Q

A plaintiff brought an action against a defendant for damages to her vehicle resulting from a car accident. To prove her damages, the plaintiff introduced a dated and authenticated invoice from the mechanic who repaired her car after the accident. After the invoice was admitted into evidence, the woman sought to enter into evidence a printout of a digital photograph of her vehicle at the scene of the accident. If admitted, she plans to testify that she called her brother immediately after the accident so he could meet her at the scene and take photographs. She will also testify that her brother took this photograph the day of the accident, and that the photograph fairly and accurately represents the condition of her vehicle and its position in the intersection after colliding with the defendant’s vehicle. Should the court rule that the photograph is admissible?

A) No, because the photograph is needlessly cumulative following the admission of the invoice.
B) No, because the plaintiff must call her brother to authenticate the photograph.
C) Yes, because a digital photograph is self-authenticating.
D) Yes, because the plaintiff has personal knowledge of the accuracy of the photograph.

A

D) Yes, because the plaintiff has personal knowledge of the accuracy of the photograph.

All tangible evidence must be authenticated. To authenticate an item, the proponent must produce sufficient evidence to support a finding that the thing is what its proponent claims it is. When reproductions (e.g., photographs, diagrams, maps, movies) are introduced into evidence, they may be authenticated by the testimony of a witness with personal knowledge that the object accurately depicts what its proponent claims it does. Here, because the plaintiff was at the scene of the accident, she has personal knowledge to support her testimony that the photograph fairly and accurately represents the condition of the vehicle and its position in the intersection after colliding with the defendant’s vehicle.

Note: Although the plaintiff has already admitted the invoice as evidence of her damages, the photograph of her vehicle at the scene of the accident could include additional evidence not available on the invoice that may be relevant to her claim for damages.

110

Q

An attorney represents a corporation in a federal securities case. As the attorney reviewed her files before court, she discovered that—despite her diligence—a memo marked “PRIVILEGED AND CONFIDENTIAL” had inadvertently been included in a folder containing public financial documents. The attorney knew that she had copied this folder and produced it in its entirety to opposing counsel during discovery. However, the memo is detrimental to her client’s case. The attorney immediately contacted opposing counsel and requested that the memo be returned to her, that all copies be destroyed, and that the information within the memo not be used at trial; she included the judge on this correspondence. Opposing counsel refused to return the memo, and informed the attorney that they did plan to use it at trial. The memo in question was from the corporation’s chief executive officer to the attorney, and contained the chief financial officer’s thoughts and questions regarding the attorney’s trial strategy. Should the court allow opposing counsel to introduce the memo into evidence at trial?

A) Yes, because the attorney waived the privilege when she disclosed the memo to opposing counsel.
B) Yes, because the memo was not privileged to begin with.
C) No, because the attorney did not waive the privilege.
D) No, because all documents from clients to attorneys are privileged.

A

C) No, because the attorney did not waive the privilege.

FRE: disclosure of a protected communication does not operate as a waiver if
(i) the disclosure was inadvertent, (ii) the holder of the privilege took reasonable steps to prevent disclosure, and (iii) the holder promptly took reasonable steps to rectify the error, including contacting the party to whom the communication was disclosed and requesting that they return, sequester, or destroy the information.

Here, the disclosure was inadvertent and that the attorney had acted diligently during discovery. Further, the facts indicate that the attorney immediately notified opposing counsel of her error. As such, the privilege was not waived by her inadvertent disclosure, making answer

111

Q

An expert witness was called by the defendant to testify in a murder trial. The expert was to testify that the defendant was not responsible for his actions due to a specific mental defect. On cross-examination, the prosecutor brought to the expert witness’s attention an authoritative book on psychological conditions, judicially noted to be a reliable authority in the field. The book described the symptoms of the mental defect at issue differently than the expert witness had described them, and the prosecutor read the book’s description into evidence. The prosecutor wanted the jury to be able to consider the book’s description as substantive evidence, but the defendant objected that the description could be used for impeachment purposes only, and not as substantive evidence. The prosecutor further wanted to introduce the book itself into evidence; the defendant objected to this as well. Should the court allow the jury to consider the description in the book as substantive evidence, and should the book itself be introduced as evidence?

A) The description should be considered for impeachment purposes only, and the book should not be introduced into evidence.
B) The description should be considered as substantive evidence, and the book should not be introduced into evidence.
C) The description should be considered as substantive evidence, and the book should be introduced into evidence.
D) The book should be introduced into evidence, though the description may be used only for impeachment purposes.

A

B) The description should be considered as substantive evidence, and the book should not be introduced into evidence.

Hearsay Exception: Treatises: A statement contained in published treatises or periodicals on a subject of history, medicine, or other science or art is admissible if
(i) the treatise is established as a reliable authority by the testimony of a witness, expert, or by judicial notice, and
(ii) an expert relied on it during direct examination or it was brought to the expert’s attention on cross-examination.

May be used for substance and impeachment

112

Q

In a pre-trial hearing, a judge determined that a defendant’s confession was given voluntarily to a police detective after the detective had given Miranda warnings to the defendant. At this hearing, the defendant testified. At trial, the defense did not contest the defendant’s receipt of Miranda warnings, but sought to question the police detective about the manner in which the defendant was interrogated after receiving the warnings in order to call into question whether the confession was voluntary. The defense does not plan to call the defendant to the witness stand. Should the court permit this line of questioning?

A) No, because there had been a judicial determination that the confession was voluntary.
B) No, because the defense did not challenge the defendant’s receipt of Miranda warnings.
C) Yes, because a party may introduce evidence that is relevant to the weight and credibility of other evidence.
D) Yes, because a defendant may testify at a hearing regarding a preliminary question without being required to testify at trial.

A

C) Yes, because a party may introduce evidence that is relevant to the weight and credibility of other evidence.

Even though a judge has decided that evidence, such as a confession, is admissible, a party may nevertheless introduce other evidence that is relevant to the weight and credibility of the admitted evidence

113

Q

A plaintiff initiated a libel action against her former boyfriend after he posted a written statement on his web site accusing the plaintiff of being able to afford a new car only because she was a con artist. At trial, the defendant called a neighbor of the plaintiff to testify that he and the plaintiff had run several cons together to earn money. The plaintiff objected to the testimony. Should the court admit this testimony for the purpose of proving that the plaintiff is a con artist?

A) No, because the introduction of specific bad acts in a civil action is prohibited.
B) No, because the testimony is inadmissible character evidence.
C) Yes, because the evidence is relevant to the defendant’s defense.
D) Yes, because character evidence must be in the form of specific acts rather than reputation or opinion testimony.

A

C) Yes, because the evidence is relevant to the defendant’s defense.

In a defamation action involving a private person and a matter that is not of public concern, as is the case here, truth is a defense. Evidence that the plaintiff has run cons would support the truthfulness of the defendant’s statement that the plaintiff is a con artist.

Note: Truth is a defense to this defamation action, and thus the plaintiff’s character is an essential element of the defense

Civil case relevant traits: negligent hiring, fraud, defamation

114

Q

A defendant on trial for forging checks took the stand in his own defense. On direct examination, the defendant denied having forged any checks; he stated that before he graduated from college the year before, he worked in his university’s academic records office, indicating that he was “a trustworthy person.” On cross-examination, the prosecutor asked the defendant if he had falsified records while working in the academic records office. The defendant denied that he had done so. The prosecutor then wanted to call to the stand his former supervisor from the university to testify that she had to investigate the defendant after allegations of misconduct, and that when questioned, he had admitted to her that he had falsified records. The defendant was removed from his position, but no formal charges had been brought against him. Should the prosecutor be allowed to call the defendant’s former supervisor to the stand to testify as to the falsified records?

A) Yes, in order to impeach the defendant and to present propensity evidence.
B) Yes, but only to impeach the defendant.
C) No, because the testimony would contain hearsay.
D) No, because the testimony would be extrinsic.

A

D) No, because the testimony would be extrinsic.

The former supervisor may not testify about the falsified records because it would be impeachment by extrinsic evidence of a specific instance of conduct. A specific instance of conduct, if used to impeach the credibility of a testifying witness, may not be proved by the introduction of extrinsic evidence. The adverse party may cross-examine the witness about the conduct, but must take his answer as he gives it.

Note: when a person is charged with one crime, extrinsic evidence of a specific instance of conduct is inadmissible to establish that the defendant had a propensity to commit that crime. Since the facts do not indicate that the prior bad act is being used as evidence for something circ*mstantial and relevant, such as motive, common plan or scheme, or identity, the supervisor’s testimony is not admissible as substantive evidence.

115

Q

A plaintiff sued a defendant for injuries she sustained when she slipped on a wet floor in the defendant’s restaurant. The plaintiff saw a physician and underwent physical therapy sessions to treat her injuries. During one session, the plaintiff said to her physical therapist, “You know, I saw the ‘Caution, Floor is Wet’ sign before I fell, but I was in such a hurry to get back to my table that I ignored it.” Another patient undergoing physical therapy with another therapist overheard the statement, and informed the defendant, who happened to be his friend. The defendant wants to introduce the testimony of his friend, as whether the plaintiff had notice of the wet floor is at issue in the case. The plaintiff objects to the testimony. Should the court allow the friend to testify as to the plaintiff’s statement?

A) No, because the statement is inadmissible hearsay.
B) No, because the statement is privileged.
C) Yes, because the statement is not hearsay.
D) Yes, because the statement was made for the purposes of medical diagnosis or treatment.

A

C) Yes, because the statement is not hearsay.

Opposing party statement: Exemption (non-hearsay)

The woman was with her physician, but did not make the statement in relation to seeking medical treatment. Thus, it is not privileged.

116

Q

In a criminal trial for attempted murder, the prosecutor seeks to introduce a statement made by the victim immediately after he was attacked by the defendant. The victim, very seriously injured, shouted the defendant’s name and said, “I can’t believe you shot me! I’m dying!” At the time of the trial, the victim has mostly recovered from his injuries, but suffered permanent memory loss, has no recollection of the incident at all, and has no recollection of making the statement. The prosecutor seeks to introduce the statement as a dying declaration, but the defendant objects. Should this statement be admissible under the “dying declaration” exception to the hearsay rules?

A) No, the statement is not admissible as a dying declaration.
B) No, because the victim did not die.
C) Yes, because the victim is unavailable due to his inability to remember.
D) Yes, because the proceeding in which the statement will be introduced is a criminal trial.

A

A) No, the statement is not admissible as a dying declaration.

Dying declaration = only available in homicide cases (or civil cases) This was an attempted murder case.

MBE Questions Flashcards by Abby Schwarz (2024)

FAQs

How many questions do you need to get right on the MBE? ›

However, the NCBE's scale means that if you get 60-66% correct, you should “pass” the MBE, depending on your jurisdiction's overall passage rate. So, to answer the question “How Many MBE Questions Do I Need to Get Correct?”, answering 60-66% correct may get you there.

How many practice MBE questions should I do? ›

For most students, this optimal range is between 800-1000 questions. This includes both untimed questions and timed sections. As you work toward completing MBE practice questions, remember that the quality of your practice is key.

Is 60% enough to pass MBE? ›

These are the approximate scores you want to aim for in order to have a passing MBE score: If you are aiming for a MBE converted score of 131-135 (which is a passing score in most jurisdictions that require scores of 262-270 to pass), you should aim for 58%-62%. We suggest aiming for at least 60% to be safe.

Is 70% good on the MBE? ›

We suggest aiming for at least 60% to be safe. If you are aiming for an MBE score of 136-140 (which is a passing MBE score in jurisdictions that require a 272-280 to pass) you should aim for 62%-65%. If you are aiming for an MBE score of 144 (which is considered passing in California), you should aim for about 67%.

How many questions can you get wrong on the MBE? ›

That means you'll need to get 115 questions correct out of 190 — or that you can miss up to 75 questions and still pass the MBE!

What is the hardest MBE subject? ›

So... the most tested MBE subjects & topics …
  • 1 - Civil Procedure. The most heavily tested Civil Procedure topics are: ...
  • 2 - Constitutional Law. The most heavily weighted topic in Constitutional Law is Individual Rights. ...
  • 3 - Contracts. ...
  • 4 - Criminal Law & Procedure. ...
  • 5 - Evidence. ...
  • 6 - Real Property. ...
  • 7 - Torts.

Are Barbri questions harder than actual MBE? ›

The Barbri invented questions are harder than the real MBE questions. (Real MBE questions are still difficult though.) The Barbri MBE compares you to how others score on the Barbri simulated MBE.

How long should 30 MBE questions take? ›

How much time you should spend on each MBE® question? A good general rule of thumb is that you have roughly 1 minute and 48 seconds per question. This is based on 200 questions in 6 hours, or 1.8 minutes per question.

How long should 25 MBE questions take? ›

Regular timing conditions

So, if you take the bar exam under normal timing conditions, you will have, on average, 1.8 minutes, or 1 minute and 48 seconds, to answer each MBE question. As a goal, try to answer, during your practice, 9 questions every 15 minutes.

Can you fail the MBE and still pass the bar? ›

The answer, in most states, is yes. In most states, as long as you make up for any lost points on the essay portion of the exam, you can fail the MBE and still pass the bar exam.

Top Articles
Latest Posts
Article information

Author: Aron Pacocha

Last Updated:

Views: 6208

Rating: 4.8 / 5 (48 voted)

Reviews: 95% of readers found this page helpful

Author information

Name: Aron Pacocha

Birthday: 1999-08-12

Address: 3808 Moen Corner, Gorczanyport, FL 67364-2074

Phone: +393457723392

Job: Retail Consultant

Hobby: Jewelry making, Cooking, Gaming, Reading, Juggling, Cabaret, Origami

Introduction: My name is Aron Pacocha, I am a happy, tasty, innocent, proud, talented, courageous, magnificent person who loves writing and wants to share my knowledge and understanding with you.